Вес топлива: Удельный вес дизельного топлива. Вес дизельного топлива в 1 литре

Содержание

Удельные веса и плотности жидких топлив. Бензин, керосин, дизтопливо, пропан, бутан, мазут. Сколько весит литр бензина.

Содержание:

  • Перевод топлива из тонн в литры и обратно дизельного, бензина и керосина
    • Формулы пересчета
    • Пересчет дизельного топлива
    • Как сориентироваться, какое число подставлять в формулу?
      • Пересчет бензинов
      • Пересчет керосинов
  • Сколько тонн и литров, в кубе дизельного топлива
    • Удельный вес дизтоплива летнего
    • Удельный вес дизтоплива зимнего
      • Формула определения веса ДТ
      • Формула определения объема ДТ
      • Формула определения плотности ДТ
  • Сколько литров в тонне дизельного топлива

Перевод топлива из тонн в литры и обратно дизельного, бензина и керосина

Топливо всех марок поставляется оптовыми компаниями в тоннах. Продается в рознице в литрах, поэтому вопросы перевода веса в объем и обратно актуальны, в основном для бухгалтеров предприятий, работающих в этом бизнесе, и налоговых служб, надзирающих за правильным начислением налогов с объемов продаж. Обычный покупатель топлива для своего автомобиля редко интересуется этими тонкостями, поскольку оплачивает и летом, и зимой литры.

Формулы пересчета

Объем и масса жидкости связаны формулой: М = V · ρ,

где М – масса жидкости в тоннах, V – ее объем в м³, ρ – плотность в т/ м³.

В реальной практике менеджеры предпочитают иметь дело с тоннами (закупка топлива) и с литрами (продажа). Если приведенную выше формулу выразить через эти величины, ее вид станет таким:

М = V · ρ /1000

  • где ρ – плотность жидкости в кг/л (числовое значение),

V – объем жидкости в литрах;

M – масса жидкости в тоннах.

Чтобы выяснить, к примеру, сколько весят 1000 л дизтоплива плотностью 0,83 кг/л, подставляем величины в формулу, получаем массу в тоннах:

М = 1000 · 0,83 / 1000 = 0,83 т.

Обратный перевод (из тонн в литры) производим по формуле V = M · 1000/ρ (масса в тоннах, плотность в кг/л, объем в литрах).

Топливо (и дизельное, и бензины, и керосины) физически не обладают постоянной плотностью — она зависит от температуры жидкости, уменьшается с ростом температуры, и растет с ее падением.

Именно поэтому перевод имеющейся массы топлива в объем для каждого значения температуры жидкости будет давать разные значения. Изменение температуры, а также возможное испарение части жидкости изменят как массу, так и, соответственно, плотность вещества. Если испарениями пренебречь, то главным действом при пересчете массы в объем и наоборот становится установление плотности жидкости.

Пересчет дизельного топлива

В практике продаж дизтоплива фигурируют различные значения плотности, используемые в разных климатических зонах как нормативные для упрощения торговли. ГОСТ № 305-82 устанавливает значения плотности при 20º С для трех видов дизтоплива — летнего (Л), зимнего (З) и арктического (А):

  • Л – 0,860 кг/л;

З – 0,840 кг/л;

А – 0,830 кг/л.

Минпромэнерго установил для дизтоплива среднее значение плотности для расчетов. Оно составляет 0,769 кг/литр. В свою очередь Ростехнадзор использует в качестве усредненного значения плотности дизтоплива величину 0,84 кг/л.

Как сориентироваться, какое число подставлять в формулу?

Федеральная налоговая служба РФ, ссылаясь на приказ Минэнерго, считает, что плотность горючего нужно устанавливать по факту при получении партии топлива замером нефтеденсиметром — специальным измерительным прибором типа ареометра.

Если прибор отсутствует, то используют средние значения плотности дизтоплива, которые можно узнать в местном отделении Ростехнадзора.

Пересчет бензинов

Плотность бензинов меняется в диапазоне 0,70 кг/л – 0,78 кг/л.

При пересчетах применяется выведенная выше формула, в которую, при отсутствии инструментального замера плотности топлива подставляют усредненное значение ρ:

  • Для АИ-80 0,715 кг/л;

АИ-92 0,735 кг/л;

АИ-95 0,750 кг/л;

АИ-98 0,765 кг/л.

Пересчет керосинов

Значения плотности керосинов меняется, в зависимости от марки, в пределах: 0,775 кг/л — 0,85 кг/л. Примеры:

  • осветительный керосин марки КО-30: плотность 0,790 кг/л;

осветительный керосин марки КО-20: плотность 0,83 кг/л;

авиационный керосин гидрированный для сверхзуковой авиации: плотность 840 кг/л. И т.д.

Пересчет керосинов из литров в тонны производится описанным выше методом после определения или установления значения плотности.

Сколько тонн и литров, в кубе дизельного топлива

Как рассчитать плотность дизельного топлива

Объем кубический метр.
В этом объёме (независимо от температуры) будет всегда 1000 (тысяча) литров или кубических дециметров.
А вот масса дизельного топлива сильно меняется от температуры. На этом навариваются на заправках, а особенно донкерманы на танкерах.
В удачный рейс на танкере 7 000 тонн можно до 25 тонн соляры налево сплавить)))) )

При повышении температуры объем нефтепродуктов увеличивается и определяется по формуле
V 2 = V1 (1 + ∆tβ) ,

где V2 – объем нефтепродукта при повышении температуры на 1 °С; V1 – первоначальный объем нефте-
продукта; ∆t – разность температур; β – коэффициент объемного расширения нефтепродукта (табл. 2).
2 Коэффициенты объемного расширения нефтепродуктов
в зависимости от плотности при +20 °С на 1 °С

Плотность, Плотность,
г/см3 β г/см3 β

0,700 …0,710 0,00127 0,800 …0,810 0,00095
0,710 …0,720 0,00123 0,800 …0,810 0,00092
0,720 …0,730 0,00120 0,800 …0,810 0,00089
0,730 …0,740 0,00116 0,800 …0,810 0,00087
0,740 …0,750 0,00113 0,800 …0,810 0,00084
0,750 …0,760 0,00110 0,800 …0,810 0,00082
0,760 …0,770 0,00107 0,800 …0,810 0,00079
0,770 …0,780 0,00104 0,800 …0,810 0,00077
0,790 …0,80 0,00098 0,800 …0,810 0,00072

В одном кубе 1000 литров или приблизительно 850 кг (последнее сильно зависит от температуры)

в один кубический метр помещается 1000л жидкости, плотность диз топлива составляет не более 860 кг/м³, отсюда можем сделать вывод, что масса топлива составит 860 кг….

= * = 0,85*1000 = 850кг=0,85т (примерно)
Куб жидкости = Тысяча литров жижкости (объём = объёму)

Р. S. Это также, как и про молоко 1 литр молока больше по объёму 1 кг молока, а 1 литр подсолнечного масла весит меньше 1 кг.

Удельный вес дизтоплива летнего

Удельный вес летнего дизтоплива напрямую зависит от его температуры. Государственным стандартом установлен в пределах 8440 Н/м3.

Удельный вес дизтоплива зимнего

Удельный вес зимнего топлива зависит от его температуры. Государственным стандартом установлен в пределах 8240 Н/м3.

Формула определения веса ДТ

Вес топлива определяется умножением плотности нефтепродукта на его объем. 1850 литров ДТ при плотности 0,840 кг/м3 будет весить 1554 кг. 1000 литров дизтоплива плотностью 0,860 кг/м3 будет весить 860 кг.

Формула определения объема ДТ

Актуальный при транспортировке, реализации и бухгалтерском учете вопрос: как перевести вес топлива в объем?

Чтобы узнать объем дизельного топлива необходимо его массу поделить на плотность. Если есть 1 тонна ДТ, а его плотность составляет 0,840 кг/м3 – объем составит 1 190 литров 476 грамм.

Формула определения плотности ДТ

Плотность дизельного топлива – это соотношение массы нефтепродукта к его объему. Если есть 860 кг дизтоплива объемом 1000 литров, то плотность составит 0,860 кг/м3.

Плотность дизельного топлива регламентируется ГОСТ 305-82. Стандарт фиксирует значение при 20 градусах по Цельсию. Плотность дизтоплива, в зависимости от его сезонного вида государственными стандартами установлена следующая:

  • зимнего – 860 кг/м3;

летнего — 840 кг/м3;

арктического – 830кг/м3.

Для определения плотности дизельного топлива другим методом нужно:

  • В паспортных данных нефтепродукта найти плотность нефтепродукта при 20 градусах по Цельсию.

Замерять фактическую температуру дизельного топлива в емкости для транспортировки или хранения.

Разность температуры умножаем на коэффициент 0,0007.

Вносим поправку.

Если температура выше – отнимаем значение от паспортной плотности, если ниже добавляем.

Сколько литров в тонне дизельного топлива

Очистка дизельного топлива своими руками

При ​температуре 20º С, удельный вес стандартного диз. топлива 0,825 кг/л. Поэтому объем тонны солярки равен 1212.12литров, при 20ºС.

1160, 1140 литров примерно, зависит от качества дизеля

Что бы это узнать, необходимо знать плотность ДТ, вот вам формула. M=pV, отсюда находим отъем V= m/p, м- масса, р- плотность ДТ

1176,5кг. (коэффициент перевода 0,85)

нормальная плотность солярки 0.8 примерно вот и считай 1000 литров равна 800 килограммам

Вес дизельного топлива в 1 литре

Сколько килограмм в литре солярки

Масса – это характеристика тела, являющаяся мерой гравитационного взаимодействия с другими телами.

Объем – это количественная характеристика пространства, занимаемого телом, конструкцией или веществом.

Плотность – это физическая величина, определяемая как отношение массы тела к объему тела.

Взаимосвязь литров и килограмм солярки определяется простой математической формулой:

V – объем;
m – масса;
p – плотность.

В расчете принята плотность солярки = 840 кг/м3.

Плотность солярки может изменяться в зависимости от температуры и давления. Точное значение плотности солярки Вы можете найти в справочниках.

Смотрите также универсальную программу перевода литров в кг для любого вещества в зависимости от его плотности.

Если необходимо перевести м3 в тонны, то смотрите программу перевода тонн в м3.

Если необходимо перевести кг в м3, то смотрите программу перевода кг в м3.

Вопрос: Сколько кг в литре солярки?

Ответ: 1 кг солярки равен 1.19 литра.

Вопрос: Сколько литров в килограмме солярки?

Ответ: 1 литр солярки равен 0.84 килограмм (кг).

Быстро решить эту простейшую математическую операцию можно с помощью нашей онлайн программы. Для этого необходимо в соответствующее поле ввести исходное значение и нажать кнопку.

На этой странице представлена самая простая программа для перевода килограммов солярки в литры. С помощью этого онлайн калькулятора вы в один клик сможете перевести литры солярки в кг и обратно.

Для начала хотелось бы уточнить, что понимается под удельным весом (далее УВ) в физике и химии, а уже потом перейти к удельному весу дизельного топлива или как его еще называют – удельный вес солярки.

Пробежимся по теории.

Удельный вес топлива

Удельный вес это отношение веса [ P ] какого-либо рассматриваемого вещества к его объему [ V ], именно веса, а не массы как многие думают. Впрочем, разницы тут особой для нас нет, это только с научной точки зрения понятия различимые и путать их никак нельзя. В быту уж так прижилось, что вес это масса.

Удельный вес вещества [ y ] можно также выразить через его плотность [ p ] : y=p*g

где g – ускорение свободного падения в конкретной точке пространства, обычно считают его равным 9,81 м/с*с.

Единицей измерения УВ является величина 1 Н/м3 (Ньютон, деленный на метр кубический).

Плотность топлива

Плотность топлива – это количество его массы в килограммах, которое помещается в одном кубическом метре. Данная величина не постоянная и зависит от температуры дизельного топлива, что плохо сказывается на работе двигателя автомобиля, если солярка по плотности плохого качества. Чем выше температура жидкости, тем меньше ее плотность и наоборот. Так же известен тот факт, что чем выше плотность автомобильного топлива, тем тяжелее его фракционный состав. Это приводит к тому, что у бензина или солярки существенно ухудшаются процессы распыления и испарения, поэтому в камерах сгорания двигателя и в топливной системе более интенсивно происходят различного рода отложения, что со временем все сильнее затрудняет передвижение топлива по системе. Так же это способствует образованию нагара на клапанах двигателя.

Удельный вес солярки

Плотность топлива и, следовательно, его удельный вес измеряют специальным прибором, который называется ареометр.

По действующему ГОСТу для удельного веса солярки приняты следующие значения (для температуры ДТ +20С):

удельный вес летнего дизельного топлива должен быть в пределах 8440 Н/м3
удельный вес зимнего дизельного топлива – 8240 Н/м3

или в плотности:

плотность летнего дизельного топлива – 860 кг/м3
плотность зимнего дизельного топлива – 840 кг/м3
плотность арктического дизельного топлива – 830 кг/м3

На практике, если брать в расчет только качественное ДТ, получается, что при изменении температуры солярки на один градус по Цельсию, ее плотность изменяется на 0,00075. Данный коэффициент можно использовать для перерасчета величины плотности ДТ в различных температурных условиях. Но стоит помнить, что на большинстве автозаправочных станций качество продукта оставляет желать лучшего, и какие примеси в нем присутствуют никому не известно. Если плотность чистого топлива и поддается перерасчету по такому коэффициенту, то плотность примесей в нем далеко не всегда.

Вес 1 литра дизельного топлива (солярки)

Исходя из приведенных выше значений плотности солярки, легко вычислить вес 1 литра дизельного топлива. Варьироваться он будет в пределах от 830 грамм до 860 грамм, то есть чем выше температура солярки, тем легче будет весить 1 ее литр.

Советуем к прочтению интересные статьи:

  • Прибор для измерения плотности
  • Как пользоваться ареометром

Компания «Ренетоп» предлагает низкую цену на дизельное топливо с доставкой по Уралу.

Удельный вес рассчитывается путем умножения плотности на коэффициент ускорения свободного падения, который всегда составляет 9,81 м/с2.

Например, 1 кг дизельного топлива плотностью 840 кг/м3 будет иметь удельный вес 8240 Н/м3.

Важную роль отыгрывает плотность дизельного топлива. Она меняется при перемене температуры топлива. При изменении температуры на 1 градус по Цельсию плотность изменяется коэффициент 0,0007. При снижении температуры на 1 градус плотность повышается, при повышении снижается.

Посмотрите наши цены:

Удельный вес дизтоплива летнего

Удельный вес летнего дизтоплива напрямую зависит от его температуры. Государственным стандартом установлен в пределах 8440 Н/м3.

Удельный вес дизтоплива зимнего

Удельный вес зимнего топлива зависит от его температуры. Государственным стандартом установлен в пределах 8240 Н/м3.

Формулы расчета плотности, веса и объема дизтоплива

Формула определения веса ДТ

Вес топлива определяется умножением плотности нефтепродукта на его объем. 1850 литров ДТ при плотности 0,840 кг/м3 будет весить 1554 кг. 1000 литров дизтоплива плотностью 0,860 кг/м3 будет весить 860 кг.

Формула определения объема ДТ

Актуальный при транспортировке, реализации и бухгалтерском учете вопрос: как перевести вес топлива в объем?

Чтобы узнать объем дизельного топлива необходимо его массу поделить на плотность. Если есть 1 тонна ДТ, а его плотность составляет 0,840 кг/м3 – объем составит 1 190 литров 476 грамм.

Формула определения плотности ДТ

Плотность дизельного топлива – это соотношение массы нефтепродукта к его объему. Если есть 860 кг дизтоплива объемом 1000 литров, то плотность составит 0,860 кг/м3.

Плотность дизельного топлива регламентируется ГОСТ 305-82. Стандарт фиксирует значение при 20 градусах по Цельсию. Плотность дизтоплива, в зависимости от его сезонного вида государственными стандартами установлена следующая:

  • зимнего – 860 кг/м3;
  • летнего – 840 кг/м3;
  • арктического – 830кг/м3.

Для определения плотности дизельного топлива другим методом нужно:

  • В паспортных данных нефтепродукта найти плотность нефтепродукта при 20 градусах по Цельсию.
  • Замерять фактическую температуру дизельного топлива в емкости для транспортировки или хранения.
  • Разность температуры умножаем на коэффициент 0,0007.
  • Вносим поправку. Если температура выше – отнимаем значение от паспортной плотности, если ниже добавляем.

Удельный вес дизельного топлива, вес дизельного топлива в 1 литре

Солярка, другое название этого продукта – дизельное топливо, представляет собой продукт жидкого типа, который используют в качестве топлива в двигателях внутреннего сгорания дизельного типа. Получается данный материал из газойлевых керосиновых фракций перегонкой нефти прямым способом.

Данный продукт предназначен для применения в качестве топливного материала в дизельных двигателях с оборотами в диапазоне от 600 до 1000 в минуту. Основными потребителями солярки являются такие виды транспорта и оборудования, как грузовой автотранспорт, железнодорожный транспорт, военная техника, водный транспорт, сельскохозяйственная техника, электрогенераторы дизельного типа, а также некоторые виды легкого автотранспорта.

Этот вид материала в разговорной речи получил свое название «солярка» благодаря немецкому слову Solaröl, которое в переводе значит – солнечное масло. Так обозначались тяжелые фракции материалов при перегонке нефти.

При работе с соляркой важно учитывать ее качество. Об этом отлично даст понять такой параметр как удельный вес солярки.

Нормативы плотности дизтоплива кг/л

Плотность дизельного топлива обозначает его массу, которая помещается в одном кубометре. Она регламентируется ГОСТ 305-82 и ГОСТ Р 52368-2005.

Нефтеперерабатывающие предприятия производят дизельное топливо, сертифицирующееся соответствующими органами по всем характеристикам. Если плотность не соответствует показателю, прописанному в государственном стандарте, дизельное топливо не получает паспорт качества и не допускается на рынок к реализации.

Плотность дизтоплива, в зависимости от марки, стандартами установлена следующая:

  • зимнего – 860 кг/м3;
  • летнего – 840 кг/м3;
  • арктического – 830кг/м3.

Повышенная плотность дизельного топлива свидетельствует о наличии в его составе тяжелых фракций, значительно утяжеляющих работу двигателя внутреннего сгорания. В результате использования топлива с повышенной плотностью на стенках камеры сгорания образуется нагар.

Плотность ДТ указывает и на количество энергии, выделяемой в результате его сгорания. Более плотное топливо выделяет большее количество энергии, что повышает коэффициент полезного действия силового агрегата. Летнее дизельное топливо, из-за большей плотности экономичнее, чем зимнее, с сравнительно низкой плотностью.

Чтобы рассчитать объем дизельного топлива в литрах нужно его массу разделить на плотность. В 1 тонне зимнего топлива будет 1163 литра ДТ, в тонне летнего – 1190. Объем дизтоплива постоянно изменяется и зависит от определенных факторов.

Доставка дизельного топлива без обмана:

Это интересно: Как проверить уровень масла в коробке dp0

Сколько весит 1 литр солярки (в кг)?

Чем выше температура жидкости, тем меньше ее плотность и наоборот. Так же известен тот факт, что чем выше плотность автомобильного топлива, тем тяжелее его фракционный состав. Это приводит к тому, что у бензина или солярки существенно ухудшаются процессы распыления и испарения, поэтому в камерах сгорания двигателя и в топливной системе более интенсивно происходят различного рода отложения, что со временем все сильнее затрудняет передвижение топлива по системе. Так же это способствует образованию нагара на клапанах двигателя.

Классификация ДТ

Сегодня в каждой стране действуют свои стандарты качества на дизельное топливо, но есть основные категории классификации, общие для любого вида солярки прописанные в межгосударственных ГОСТах 32511-2013(EN 590:2009), 305-2013 и 2517-2012.

Выделяют топливо с низкой вязкостью (дистиллятное), применяемое для высокооборотисных двигателей и остаточное, с низкой вязкостью. В отличие от гидроочищенных керосино-газойлевых фракций дисцилятного топлива, остаточное состоит из смеси мазутов и керосино-газойлевых фракций.

Главный принцип классификации – сезонность.

  • Летнее ДТ.

Его можно использовать только при плюсовых температурах. Удельный вес летнего дизеля – 860 кг/м3 (определяется теоретическая плотность любого вида ДТ при +20ºС). С повышением температуры удельный вес уменьшается ориентировочно на 0,0007 г/см3, а при понижении температуры, соответственно, увеличивается.

При температуре –5ºС парафины летнего ДТ густеют и забивают систему. Использование присадок помогает отсрочить загустевание, но повышенную при минусовых температурах плотность летнего топлива они не изменяют. Температура вспышки 62ºС.

  • Зимнее ДТ.

Рекомендованная температура эксплуатации от –20ºС до –35ºС. Удельный вес зимней солярки 840 кг/м3. При –35 градусах Цельсия застывает. Вспышка происходит при +40 ºС.

  • Арктическое ДТ.

Рекомендуется использовать в диапазоне отрицательных температур 45–50ºС. Плотность дизеля 830 кг/м3, а температура вспышки +35 градусов Цельсия. Получают ДТ А путем депарафинизации летнего ДТ либо добавлением в чистый керосин повышающие цетановое число присадок и моторного масла для повышения смазывающих свойств.

Вес дизельного топлива в 1л составляет примерно 850 г или 0.85 кг
Таблица веса дизельного топлива

Вид ДТТемпературный диапазон (ºС)Плотность (кг/м3)Удельный вес (г/см3)
Летнее+0 …8600,86
Зимнее–35… –208400,84
Арктическое–50…–458300,83

Перевод топлива из тонн в литры и обратно дизельного, бензина и керосина

Топливо всех марок поставляется оптовыми компаниями в тоннах. Продается в рознице в литрах, поэтому вопросы перевода веса в объем и обратно актуальны, в основном для бухгалтеров предприятий, работающих в этом бизнесе, и налоговых служб, надзирающих за правильным начислением налогов с объемов продаж. Обычный покупатель топлива для своего автомобиля редко интересуется этими тонкостями, поскольку оплачивает и летом, и зимой литры.

Формулы пересчета

Объем и масса жидкости связаны формулой: М = V · ρ,

где М – масса жидкости в тоннах, V – ее объем в м³, ρ – плотность в т/ м³.

В реальной практике менеджеры предпочитают иметь дело с тоннами (закупка топлива) и с литрами (продажа). Если приведенную выше формулу выразить через эти величины, ее вид станет таким:

М = V · ρ /1000

  • где ρ – плотность жидкости в кг/л (числовое значение),

V – объем жидкости в литрах;

M – масса жидкости в тоннах.

Чтобы выяснить, к примеру, сколько весят 1000 л дизтоплива плотностью 0,83 кг/л, подставляем величины в формулу, получаем массу в тоннах:

М = 1000 · 0,83 / 1000 = 0,83 т.

Обратный перевод (из тонн в литры) производим по формуле V = M · 1000/ρ (масса в тоннах, плотность в кг/л, объем в литрах).

Топливо (и дизельное, и бензины, и керосины) физически не обладают постоянной плотностью — она зависит от температуры жидкости, уменьшается с ростом температуры, и растет с ее падением.

Именно поэтому перевод имеющейся массы топлива в объем для каждого значения температуры жидкости будет давать разные значения. Изменение температуры, а также возможное испарение части жидкости изменят как массу, так и, соответственно, плотность вещества. Если испарениями пренебречь, то главным действом при пересчете массы в объем и наоборот становится установление плотности жидкости.

Пересчет дизельного топлива

В практике продаж дизтоплива фигурируют различные значения плотности, используемые в разных климатических зонах как нормативные для упрощения торговли. ГОСТ № 305-82 устанавливает значения плотности при 20º С для трех видов дизтоплива — летнего (Л), зимнего (З) и арктического (А):

  • Л – 0,860 кг/л;

З – 0,840 кг/л;

А – 0,830 кг/л.

Минпромэнерго установил для дизтоплива среднее значение плотности для расчетов. Оно составляет 0,769 кг/литр. В свою очередь Ростехнадзор использует в качестве усредненного значения плотности дизтоплива величину 0,84 кг/л.

Как сориентироваться, какое число подставлять в формулу?

Федеральная налоговая служба РФ, ссылаясь на приказ Минэнерго, считает, что плотность горючего нужно устанавливать по факту при получении партии топлива замером нефтеденсиметром — специальным измерительным прибором типа ареометра.

Если прибор отсутствует, то используют средние значения плотности дизтоплива, которые можно узнать в местном отделении Ростехнадзора.

Пересчет бензинов

Плотность бензинов меняется в диапазоне 0,70 кг/л – 0,78 кг/л.

При пересчетах применяется выведенная выше формула, в которую, при отсутствии инструментального замера плотности топлива подставляют усредненное значение ρ:

  • Для АИ-80 0,715 кг/л;

АИ-92 0,735 кг/л;

АИ-95 0,750 кг/л;

АИ-98 0,765 кг/л.

Пересчет керосинов

Значения плотности керосинов меняется, в зависимости от марки, в пределах: 0,775 кг/л — 0,85 кг/л. Примеры:

  • осветительный керосин марки КО-30: плотность 0,790 кг/л;

осветительный керосин марки КО-20: плотность 0,83 кг/л;

авиационный керосин гидрированный для сверхзуковой авиации: плотность 840 кг/л. И т.д.

Пересчет керосинов из литров в тонны производится описанным выше методом после определения или установления значения плотности.

Cколько весит 1 литр дизельного топлива?

Но это факты, а что можно узнать о бензине, если взглянуть на цифры. Например, сколько весит литр бензина? И можно ли его измерить? Безусловно, знание этих чисел может открыть взгляд на эту жидкость совсем по-другому.

Как известно бензин делиться на несколько категорий, а именно: А-76, А-80, А-92, А-95 и А-98. Следует заметить, что вес одного литра бензина в каждом случае может отличаться. Итак, нам известны некоторые данные по весу жидкости в объеме одного литра, каждой из марки бензина.

  • В среднем вес одного литра бензина А-76 составляет 0,730 кг. Но это при температуре +20°С.
  • В среднем вес одного литра бензина А-80 составляет 0,730 кг.
  • В среднем вес одного литра бензина А-92 составляет 0,760 кг.
  • В среднем вес одного литра бензина А-95 составляет 0,750 кг.
  • В среднем вес одного литра бензина А-98 составляет 0,780 кг.

Итак, мы узнали, сколько же будет весить литр бензина каждой из вышеперечисленных марок. Но известно и о другой характерности бензина. Так, вес одного литра бензина напрямую будет зависеть от температуры окружающей среды. Более того, чем выше температура на градуснике, тем легче имеет вес литр бензина. В некоторых лабораториях осуществляют специальные измерения бензина. Например, при температуре +16°С плотность бензина А-92 имеет 0,765 г/л. Если отталкиваться от измерений, приведенных выше, то получаем вес одного литра бензина тяжелее. Почему? Все дело зависит как раз таки от температуры окружающей среды. По этой причине в каждом отдельном измерении вес может отличаться. Совсем другое дело, если осуществлять такие измерения, следовательно, по мере повышения или понижения температуры. Тогда мы сможем знать точную плотность бензина.

Но сегодня никто не покупает бензин по одному литру. Например, некоторые сразу заправляют полный бак своего авто. При этом аппетит того или иного автомобиля бывает разным. Другие же покупают бензин в канистрах. Например, если у вас есть канистра вместительностью 19 литров, то можно точно рассчитать вес пустой канистры и полной. Канистра, заполненная бензином, имеет вес 16,325 килограмм. Соответственно пустая канистра без содержимого имеет массу 2,550 килограмм. При этом, если брать из расчета что один литр весит 0,725 кг, то следует выполнить простое вычисление. От суммы веса канистры с бензином (16,325 кг) отнимаем сумму пустой канистры (2,550 кг). В результате мы получаем сумму 13,775 кг бензина в девятнадцати литровой канистре. Проверить эту сумму легко, если совершить следующий расчет:

13,775 кг / — 0,725 кг = 19 литров.

Итак, при помощи таких математических действий можно в точности вычислять массу бензина в той или иной канистре.

Но кто-то, возможно, возразит, зачем переводить единицу измерения литр в килограммы или даже в тонны? На самом деле это осуществляется непросто ради забавы. Для большого количества потребителей горюче-смазочных веществ это является одной из важных задач, а особенно для бухгалтеров того или иного предприятия. В процессе учета хранения то ли сыпучих, то ли жидких веществ, в нашем случае бензин зачастую возникает потребность в конвертации. Итак, пересчет объема в массу крайне важен и даже необходим для того, чтобы легче было осуществлять заполнение отчетной документации, финансовых расчетов, а также осуществления платежей при оптовых продажах топлива. Главным образом это связано с тем, что общепринятой формой поставки ГСМ и топлива является цистерна, которая имеет фиксированную вместимость. Но, учет всегда осуществляется в единицах массы! Тем более, если осуществляется оптовая продажа бензина, то вести подсчет в тоннах гораздо легче. Вот такая получается арифметика с подсчетом веса бензина!

Для начала хотелось бы уточнить, что понимается под удельным весом (далее УВ) в физике и химии, а уже потом перейти к удельному весу дизельного топлива или как его еще называют – удельный вес солярки.

Пробежимся по теории.

Формулы расчета плотности, веса и объема дизтоплива

Формула определения веса ДТ

Вес топлива определяется умножением плотности нефтепродукта на его объем. 1850 литров ДТ при плотности 0,840 кг/м3 будет весить 1554 кг. 1000 литров дизтоплива плотностью 0,860 кг/м3 будет весить 860 кг.

Формула определения объема ДТ

Актуальный при транспортировке, реализации и бухгалтерском учете вопрос: как перевести вес топлива в объем?

Чтобы узнать объем дизельного топлива необходимо его массу поделить на плотность. Если есть 1 тонна ДТ, а его плотность составляет 0,840 кг/м3 – объем составит 1 190 литров 476 грамм.

Формула определения плотности ДТ

Плотность дизельного топлива – это соотношение массы нефтепродукта к его объему. Если есть 860 кг дизтоплива объемом 1000 литров, то плотность составит 0,860 кг/м3.

Плотность дизельного топлива регламентируется ГОСТ 305-82. Стандарт фиксирует значение при 20 градусах по Цельсию. Плотность дизтоплива, в зависимости от его сезонного вида государственными стандартами установлена следующая:

  • зимнего – 860 кг/м3;
  • летнего — 840 кг/м3;
  • арктического – 830кг/м3.

Для определения плотности дизельного топлива другим методом нужно:

  • В паспортных данных нефтепродукта найти плотность нефтепродукта при 20 градусах по Цельсию.
  • Замерять фактическую температуру дизельного топлива в емкости для транспортировки или хранения.
  • Разность температуры умножаем на коэффициент 0,0007.
  • Вносим поправку. Если температура выше – отнимаем значение от паспортной плотности, если ниже добавляем.

Удельный вес дизельного топлива. Вес дизельного топлива в 1 литре

Сколько кг в 1 литре дизельного топлива. Литровые нормы. Мерный инструмент для определения без весов – литровая банка. Сколько объемный вес дизельного топлива в одном литре – кг/литр, при нормальной его плотности в обычных условиях, качество, химических состав и физические свойства заявлены в ГОСте, соответствуют ТУ производителя.
Замечания, интересные пояснения к вопросу «сколько весит литровый объем» и некоторая дополнительная информация к справочным данным по физическим свойствам.
У нас есть по факту 1 литр дизельного топлива, литровая банка или некоторое количество известное в литрах, мы хотим узнать массу: сколько его вес в кг или граммах. То есть, наша задача определить, сколько он весит: пересчитать литры дизельного топлива – единицы измерения объема в килограммы – единицы измерения веса дизельного топлива. Определив объемную массу 1 литра дизельного топлива, сколько в килограммах, без взвешивания на весах. Узнать сколько он весит (например: литровая банка), основываясь на расчетных, теоретических, справочных данных из таблицы физических свойств: удельном весе и плотности дизельного топлива. Лучше конечно пользоваться не истинной, а объемной плотностью, если удается найти объемную плотность в справочнике. Что на самом деле не так просто сделать на практике. Крайне неудобно самостоятельно пересчитывать исходя из физических свойств, еще и потому, что большинство таблиц, ГОСТов, ТУ приводит значения удельного веса и плотности «привязанные» не к литровой банке, в других единицах: тонны/м3, кг/м3, кг/куб, гр/см3. При желании можно потратить много времени, найти данные, самостоятельно выполнить пересчет и узнать, сколько будет кг в литровом объеме. Однако, такой вариант узнать сколько весит литровый объем, не для всех оказывается простым и удобным способом, тем более что он требует поисков дополнительной информации по объемной плотности дизельного топлива и его удельному весу, плюс знания формулы пересчета. Поэтому, мы сделали сами такой перерасчет на основе физических свойств приведенных в ГОСТе и ТУ, указали массу 1 литра дизельного топлива в кг, литровой банки, приведя ее в отдельной таблице весов и объемов. Выписка из общей таблицы приводится выше. Вопрос: сколько кг в одном литре дизельного топлива, литровый объем. Ответ: объемную плотность смотрите в таблице 1. Как узнать сколько весит — измеряем массу дизельного топлива литровой банкой, самостоятельно, без взвешивания на весах. На нашем сайте вы сможете найти другую интересную практическую информацию по плотности и удельному весу не указанную в обычных справочниках, таблицах, ГОСТах и ТУ, которую не так просто найти в интернете. Например, для более привычных нам единиц измерения объема: см3, м3, куба, кубометра, кубического метра. Бытовых и столовых приборов измерения продуктов: чайной ложечки, столовой ложки, стакана. Более строительных, садовых, производственных и промышленных: лопаты (штыковой и совковой), пачки, ведра, мешка, поддона, бочки, строительной или садовой тачки. Транспортных: авто цистерны, железнодорожной цистерны, железнодорожного вагона, бортовой машины, грузовика, самосвала. Это так называемые, не стандартные емкости, часто используемые на практике. Не смотря на все разнообразие емкостей, каждая из них имеет собственный литраж. А это означает, что можно пересчитать массу груза в кг через указанные в таблице 1 значения. Естественно, что такая методика имеет собственные погрешности. При этом, ошибка расчетов массы большого литража будет выше, чем погрешность определения массы литрового объема указанная в таблице 1.

© ЧП Колесник 2010-2011
Наш адрес: Днепропетровск, ул. Карла Либкнехта 57 Телефон по Украине: (063) 796-79-32 или (063) 796-19-32

Физические характеристики дизеля

Дизельное топливо относится к продуктам, полученным после перегона нефти на специальных предприятиях (НПЗ). Качество и состав готовой жидкости должны удовлетворять строгим нормативам. Значение плотности является параметром, который участвует в определении продуктивной работоспособности топлива при различных условиях.

Важно знать, что плотность демонстрирует количество килограммов жидкости в одном кубическом метре.

Специалисты знают, что данный параметр является не постоянным и зависит от внешних факторов, главным из которых является окружающая температура. Поднятие столбика термометра стимулирует уменьшение плотности, а обратный процесс повышает удельный вес дизельного топлива.

Для получения конкретного значения используется измерительный аппарат – ареометр. В процессе измерения агрегат нужно опустить в емкость с соляркой. Чтобы проводить замеры в разных жидкостях применяют различные типы ареометров. Измерения в нефтепродуктах осуществляются моделями АН, АНТ-1 или АНТ-2.

Ареометр изготовлен в виде стеклянной трубочки, внутри которой имеется градуированная вертикальная шкала. Степень бо́льшая погружения демонстрирует меньшую плотность и наоборот.

Увеличенный удельный вес жидкости является следствием того, что в ней присутствуют тяжелые углеводородные фракции. Качественная работа ДВС из-за этого может снизиться, ведь ухудшается испаряемость жидкости и не обеспечивается хорошая ее распыляемость форсунками. Дополнительный негатив от наличия большого числа тяжелых частиц в том, что на рабочих поверхностях образуется нагар и различные отложения.

Это интересно: Как происходит воспламенение топлива в дизельном двигателе

Удельный вес дизтолпива

Для бытового применения и каких-то самостоятельных работ, вопрос может задаваться иначе, когда спрашивают не вес 1 литра дизельного топлива, а сколько весит литровая банка (баночка). Обычно интересует, сколько грамм или килограмм в литровой банке. Найти такие данные: сколько весит, в интернете не так просто, как кажется. Дело в том, что общепринятый формат подачи материала в любых справочниках, таблицах, ТУ и ГОСТе, сводится к приведению только плотности и удельного веса дизельного топлива. При этом указанными единицами измерения являются один м3, куб, кубометр или кубический метр. Реже 1 см3. А нас интересует, сколько весит литровый объем. Что приводит к необходимости дополнительного пересчета кубических метров (м3) в литры. Это неудобно, хотя и возможно сделать правильный пересчет кубов в количество литров самостоятельно. Пользуясь соотношением: 1 м3 = 1000 л. Для удобства посетителей сайта, мы самостоятельно сделали перерасчеты и указали, сколько весит один литр дизельного топлива в таблице 1. Зная вес 1 литра дизельного топлива, вы не только определяете массу литровой банки, но и легко можете рассчитать, сколько весит любая другая емкость, для которой известен литраж. При этом, нужно понимать нежелательность и невозможность точных оценок сделанных на основании подобных пересчетов для больших емкостей со значительным объемом литража. Дело в том, что при таких методиках расчета возникает большая погрешность, приемлемая только в смысле приблизительной оценки массы. Поэтому, профессионалы пользуются специальными таблицами, в которых указано, сколько весит, например автомобильная или железнодорожная цистерна, бочка. С другой стороны, для прикладных и бытовых целей, для домашних условий, метод расчета исходя из литрового объема, вполне пригоден и может применяться на практике. В тех случаях, когда нам нужны более точные данные, например: при лабораторных исследованиях, для проведения экспертизы, для отладки производственного процесса, наладки оборудования и так далее. Вес 1 литра дизельного топлива лучше определять экспериментальным путем, через взвешивание на точных весах, по специальной методике, а не пользоваться справочными, теоретическими, табличными средними данными о плотности и его удельном весе.

© ЧП Колесник 2010-2011
Наш адрес: Днепропетровск, ул. Карла Либкнехта 57 Телефон по Украине: (063) 796-79-32 или (063) 796-19-32

На сегодняшний день самым потребляемым продуктом в мире является бензин. И это не удивительно. Но удивительно другое. Из истории известно, что бензин однажды использовался в медицинских целях! Невероятно, но факт. Более того, именно бензин обладает самым большим товарооборотом в мире, среди всех возможных товаров народного потребления. Именно бензин потребляется больше всего человечеством. Только представьте, он стоит на первом месте по товарообороту, а на втором кофе!

Примеры: как перевести кубы в литры?

Рассмотрим теперь обратные задачи о нахождении количества кубов в указанном количестве литров.

Задача #1 : Сколько кубов в 100 литрах? Решение : 100 * 0,001 = 0,1 куб. метр. Ответ : 100 литров – это 0,1 метра куб.Задача #6 : Сколько кубов в 1500 литрах? Решение : 1500 * 0,001 = 1,5 метров кубических. Ответ : в 1500 литров 1,5 метров кубических.
Задача #2 : Сколько кубов в 200 литрах? Решение : 200 * 0,001 = 0,2 куб. метра. Ответ : в 200 литров 0,2 м. метра.Задача #7 : Сколько кубов в 3000 литрах? Решение : 3000 * 0,001 = 3 метров кубических. Ответ : в 3000 литрах — 3 метров кубических.
Задача #3 : Сколько кубов в 140 литрах? Решение : 140 * 0,001 = 0,14 кубометров. Ответ : в 140 литров 0,14 кубометров.Задача #8 : Сколько кубов в 5000 литрах? Решение : 5000 * 0,001 = 5 метров кубических. Ответ : в 5 000 литрах — 5 метров кубических.
Задача #4 : Сколько кубов в 500 литрах? Решение : 500 * 0,001 = 0,5 кубов. Ответ : в 500 литров 0,5 кубов.Задача #9 : Сколько кубов в 10 000 литрах? Решение : 10 000 * 0,001 = 10 куб. м. Ответ : в 10 000 литрах – 10 куб. м.
Задача #5 : Сколько кубов в 1000 литрах? Решение : 1000 * 0,001 = 1 метр кубический. Ответ : в 1000 литрах 1 метр кубический.Задача #10 : Сколько кубов в 30 000 литрах? Решение : 30 000 * 0,001 = 30 куб. м. Ответ : в 30 000 литров 30 куб. м.

Для быстроты расчетов предлагаем воспользоваться вам нашими онлайн калькуляторами:

Перейдите по соответствующим ссылкам и воспользуйтесь инструкциям.

Всем, кто хорошо учился в школе и понимал физику, известно, что объем любого жидкого вещества может зависеть от двух параметров – массы и плотности. Если известны эти величины, то его можно рассчитать. Зная плотность, мы можем определить, сколько литров дизельного топлива в 1 тонне. Однако стоит при этом отметить, что плотность солярки не может быть стандартной – она варьируется в зависимости от температуры окружающей среды. Поэтому зимой в одной тонне будет находиться меньше литров топлива, летом – больше.

Вес 1 литра дизельного топлива (солярки)

Исходя из приведенных выше значений плотности солярки, легко вычислить вес 1 литра дизельного топлива. Варьироваться он будет в пределах от 830 грамм до 860 грамм, то есть чем выше температура солярки, тем легче будет весить 1 ее литр.
Сколько весит 1 куб дизельного топлива ДТ, вес 1 м3 дизельного топлива ДТ. Количество килограмм в 1 кубическом метре, количество тонн в 1 кубометре, кг в 1 м3. Объемная плотность дизельного топлива ДТ удельный вес.
Что мы хотим узнать сегодня узнать? Сколько весит 1 куб дизельного топлива ДТ, вес 1 м3 дизельного топлива ДТ ?

Нет проблем, можно узнать количество килограмм или количество тонн сразу, масса (вес одного кубометра, вес одного куба, вес одного кубического метра, вес 1 м3) указаны в таблице 1. Если кому-то интересно, можно пробежать глазами небольшой текст ниже, прочесть некоторые пояснения. Как измеряется нужное нам количество вещества, материала, жидкости или газа? За исключением тех случаев, когда можно свести расчет нужного количества к подсчету товара, изделий, элементов в штуках (поштучный подсчет), нам проще всего определить нужное количество исходя из объема и веса (массы). В бытовом отношении самой привычной единицей измерения объема для нас является 1 литр. Однако, количество литров, пригодное для бытовых расчетов, не всегда применимый способ определения объема для хозяйственной деятельности. Кроме того, литры в нашей стране так и не стали общепринятой «производственной» и торговой единицей измерения объема. Один кубический метр или в сокращенном варианте — один куб, оказался достаточно удобной и популярной для практического использования единицей объема. Практически все вещества, жидкости, материалы и даже газы мы привыкли измерять в кубометрах. Это действительно удобно. Ведь их стоимость, цены, расценки, нормы расхода, тарифы, договора на поставку почти всегда привязаны к кубическим метрам (кубам), гораздо реже к литрам. Не менее важным для практической деятельности оказывается знание не только объема, но и веса (массы) вещества занимающего этот объем: в данном случае речь идет о том сколько весит 1 куб (1 кубометр, 1 метр кубический, 1 м3). Знание массы и объема, дают нам довольно полное представление о количестве.

Плотность разных видов солярки

Все дизельное топливо делится на разные виды и не является стандартным. Как минимум существуют три вида: летнее, зимнее, арктическое топливо. Плотность при этом отличается, и чтобы посчитать, сколько литров дизельного топлива в 1 тонне, нам необходимо узнать данное значение.

Итак, летнее дизтопливо имеет плотность 860 кг/м 3 . Эта солярка отличается своим температурным диапазоном работы. При 10 градусах выше нуля данное топливо может начать застывать. Температура, при которой возможна вспышка, – 45 градусов выше нуля. Обычно оно продается в жарких странах.

Зимняя солярка имеет плотность 840 кг/м 3 . Она не застывает при температуре не ниже -35 градусов. Следовательно, данное топливо эффективно зимой. Воспламенение возможно при 40 градусах по Цельсию.

Есть также арктическое дизтопливо, которое редко встречается на заправочных станциях в городе. Его плотность – 830 кг/м 3 . Воспламенение его возможно при 35 градусах по Цельсию.

Если данный продукт продают большими объемами, расчет производится в тоннах, а не в литрах. Это объясняется тем, что жидкость может сжиматься или расширяться при низких и высоких температурах соответственно. Следовательно, объем будет расти или уменьшаться, однако масса при этом будет постоянной. Конечно, можно его покупать и в литрах, но тогда может быть высокая погрешность в лучшую или худшую сторону для покупателя или продавца.

Удельный вес дизельного топлива (солярки)

Посетители сайта, спрашивая сколько весит 1 куб, часто указывают конкретные единицы массы, в которых им хотелось бы узнать ответ на вопрос. Как мы заметили, чаще всего хотят узнать вес 1 куба ( 1 кубометра, 1 кубического метра, 1 м3) в килограммах (кг) или в тоннах (тн). По сути, нужны кг/м3 или тн/м3. Это тесно связанные единицы определяющие количество. В принципе возможен довольно простой самостоятельный пересчет веса (массы) из тонн в килограммы и обратно: из килограммов в тонны. Однако, как показала практика, для большинства посетителей сайта более удобным вариантом было бы сразу узнать сколько килограмм весит 1 куб (1 м3) дизельного топлива ДТ или сколько тонн весит 1 куб (1 м3) дизельного топлива ДТ

, без пересчета килограмм в тонны или обратно — количества тонн в килограммы на один метр кубический (один кубометр, один куб, один м3). Поэтому, в таблице 1 мы указали сколько весит 1 куб ( 1 кубометр, 1 метр кубический) в килограммах (кг) и в тоннах (тн). Выбирайте тот столбик таблицы, который вам нужен самостоятельно. Кстати, когда мы спрашиваем сколько весит 1 куб ( 1 м3), мы подразумеваем количество килограмм или количество тонн. Однако, с физической точки зрения нас интересует плотность или удельный вес. Масса единицы объема или количество вещества помещающегося в единице объема — это объемная плотность или удельный вес. В данном случае объемная плотность и удельный вес дизельного топлива ДТ. Плотность и удельный вес в физике принято измерять не в кг/м3 или в тн/м3, а в граммах на кубический сантиметр: гр/см3. Поэтому в таблице 1 удельный вес и плотность (синонимы) указаны в граммах на кубический сантиметр (гр/см3)
Таблица 1. Сколько весит 1 куб дизельного топлива ДТ, вес 1 м3 дизельного топлива ДТ. Объемная плотность и удельный вес в гр/см3. Сколько килограмм в кубе, тонн в 1 кубическом метре, кг в 1 кубометре, тн в 1 м3.

Пересчитать, узнать объемный вес: физические свойства.Величины.Количество кг в 1 литре, кг/литр.Для расчетов использовались справочные данные из:Теперь вы можете узнать сколько весит при помощи такого инструмента, как:
Погрешность измерений.
Сколько кг вес 1 литра дизельного топлива — литровая банка.Используем справочные данные по плотности и удельному весу, рассчитывая по формуле получаем объемный вес.0.821 — 0.861Справочник физических свойств, ГОСТ, ТУ.Литровая банка.до 5%
Сколько кг в 1 литре дизельного топлива, литровая банка. Для тех кого интересуют нестандартные емкости и объемы: м3, куба, кубометра, кубического метра, чайной ложечки, столовой ложки, стакана, лопаты, пачки, ведра, мешка, поддона, бочки, строительной или садовой тачки, авто цистерны, железнодорожной цистерны, железнодорожного вагона, бортовой машины, грузовика, самосвала.

Таблица 1.

‎App Store: Steer: расход топлива и сервис

Снимки экрана (iPhone)

Описание

Отслеживайте расход топлива, затраты и следите за ТО автомобиля

Отслеживайте расход и анализируйте качество топлива на различных АЗС. Не забывайте проводить ТО автомобиля. Следите за финансовыми расходами и многое другое в Steer.

Управлять несколькими транспортными средствами стало намного проще:

• Расход топлива:
Храните все данные о заправках в приложении. Steer рассчитает расход вашего автомобиля, составит вам графики и покажет какой расход топлива был на каждой из АЗС.

• ТО
Замена масла, шиномонтаж, фильтры, проверка тормозов и множество других услуг вы можете хранить в приложении Steer. Вы можете указать цену, одометр прохождения, дату и выбрать через сколько километров или времени хотите пройти ТО снова.

• Напоминания
Автомобильное приложение Steer так же помогает водителю, напоминая о регулярных услугах и ТО, напоминает о том, что пора сохранить данные о заправке, когда вы посетили АЗС.

• Синхронизация
Теперь Steer может хранить ваши данные в облачном хранилище. Это очень полезно, если вы хотите синхронизировать данные между несколькими девайсами или собираетесь восстановить данные своего аккаунта.

Приятного использования.
Помогите нам усовершенствовать приложение, оставляя отзывы после каждого обновления.

Подписка на полный доступ или одноразовая покупка не обязательна для использования всех функций приложения.
Вы можете начать с 7-дневной бесплатной пробной версии, затем 0,99 $ в месяц или 9,99 $ в год, или приобрести единожды 19,99 $ и избавиться от всех ограничений на автомобили, техническое обслуживание и количество заправок в месяц и получите доступ ко всем функциям PRO в приложении.
Вы можете управлять подпиской и отменить ее в настройках учетной записи App Store в любое время. Подписка будет обновлена в течение 24 часов до окончания периода подписки.
Условия и политика конфиденциальности — https://admin.steerapp.net/privacy-policy

Версия 3.13.1

Какие новости:
• Предполагаемый пробег для технического обслуживания
• Новый экран приветствия
• Улучшено получение типов техобслуживания из API.
• Обновления интерфейса ввода техобслуживания

Оценки и отзывы

Оценок: 81

Пока 4)

Только начал пользоваться прогой. Ещё не разобрался в ней. Но сразу столкнулся с нехваткой ТИПОВ обслуживания. Неплохо бы иметь возможность добавлять «тип обслуживания» самостоятельно, чтобы видеть, когда и какую (какой фирмы-производителя) з/часть менял, сколько стоила.

Хорошо, но есть недостатки.

Не хватает: светлой темы, экспорта отчётов и выбора вида топлива (газ/бензин).

Пропаганда

Очень странно заходить в приложение и видишь сообщения о вторжении на окраину …зачем писать бред !!!

Разработчик Nikita Marchenko указал, что в соответствии с политикой конфиденциальности приложения данные могут обрабатываться так, как описано ниже. Подробные сведения доступны в политике конфиденциальности разработчика.

Данные, используе­мые для отслежи­вания информации

Следующие данные могут использоваться для отслеживания информации о пользователе в приложениях и на сайтах, принадлежащих другим компаниям:

Связанные с пользова­телем данные

Может вестись сбор следующих данных, которые связаны с личностью пользователя:

Не связанные с пользова­телем данные

Может вестись сбор следующих данных, которые не связаны с личностью пользователя:

  • Контактные данные
  • Данные об использова­нии
  • Диагностика

Конфиденциальные данные могут использоваться по-разному в зависимости от вашего возраста, задействованных функций или других факторов. Подробнее

Информация

Провайдер
Nikita Marchenko

Размер
39,2 МБ

Категория
Утилиты

Возраст
4+

Copyright
© 2018 Marchenko Nikita

Цена
Бесплатно

  • Поддержка приложения
  • Политика конфиденциальности

Поддерживается

Другие приложения этого разработчика

Вам может понравиться

Как снизить расход топлива — эксперты объясняют — журнал За рулем

Для этого нужно прежде всего разобраться с тем, почему двигатель стал поджирать бензин.

Материалы по теме

Еще один способ сэкономить на бензине — мнение эксперта

Когда визиты на АЗС становятся подозрительно частыми, невольно начинаешь прикидывать: с чего бы вдруг? Ничего не поменялось — ни погода, ни маршруты, а расход явно вырос. Можно, конечно, съездить на сервис, но там на радостях такого порасскажут, что никакой зарплаты не хватит. Нет, предварительную диагностику лучше провести самому. Да и на сервисе будет легче беседовать.

Двигатель и его системы

Механическая часть мотора подбрасывает нам две основные проблемы, увеличивающие расход горючего, - низкую компрессию в цилиндрах и нарушенные фазы газораспределения.

Двигатель с низкой компрессией неэффективно сжигает топливо, а потому требует его много. Компрессию нужно померить — тут диагностика простая.

Материалы по теме

Штрафы привяжут к МРОТ (или другому расчетному показателю)Новый побор для водителей в 2022Возврат балльной системы для нарушителей ПДД — самое время?

Материалы по теме

6 кроссоверов, которые можно заправлять 92-м (официально!). А надо ли?

С фазами газораспределения чуть сложнее: правильность их установки оценивают по меткам, число и расположение которых зависят от типа двигателя. На современных моторах при сбитых метках обычно загорается индикатор Check Engine.

Система питания увеличивает топливный аппетит, как правило, вследствие загрязнения топливного или воздушного фильтра. Тут всё ясно: надо оценить их состояние и при необходимости заменить.

Другая возможная причина прожорливости связана с недостаточным давлением, которое создает бензонасос. Его нужно измерить манометром, после чего принять решение о дальнейших действиях. Ориентировочно давление составляет около 3,5 бар на современных автомобилях с распределенным впрыском топлива и около 3 бар на более старых моделях.

За экономию на замене свечей приходится расплачиваться на бензоколонке.

За экономию на замене свечей приходится расплачиваться на бензоколонке.

Наконец, виноватыми могут быть грязные топливные форсунки — такие не обеспечат правильного факела распыла топлива. Можно попробовать очиститель топливной системы, заливаемый в бак при заправке, но лучше всего промыть их на специализированном сервисе.

Система управления двигателем — это вообще Клондайк по части всевозможных неисправностей, повышающих расход топлива. Любой датчик, начинающий дурить, тут же увеличивает прожорливость мотора. Облегчает диагностику загорание индикатора Check Engine.

Проверить, что произошло, можно с помощью диагностического адаптера ELM327 и смартфона с приложением. Отметим и необходимость своевременной замены свечей, и использование только разрешенных производителем оригинальных свечей или аналогов проверенных производителей.

Низкое давление топлива не обеспечит правильного распыла форсунками.

Низкое давление топлива не обеспечит правильного распыла форсунками.

Выхлопная система с механическими повреждениями увеличивает расход топлива.

Выхлопная система с механическими повреждениями увеличивает расход топлива.

Система выпуска повышает расход простейшим способом — затрудняя выход отработавших газов. Это может происходить при помятых выпускных трубах, отвалившихся перегородках в глушителе, а также при неисправностях в каталитическом нейтрализаторе. С диагностикой поможет манометр, устанавливаемый вместо первого лямбда-зонда. Противодавление даже на больших оборотах двигателя не должно превышать 0,3 бара.

Система охлаждения повышает расход топлива при любом отклонении от рабочей температуры — как в плюс, так и в минус. Чтобы убедиться в правильности ее работы, штатного термометра может не хватить, да и не на всех машинах он присутствует. Проще подключить диагностический адаптер ELM327.

Трансмиссия

Материалы по теме

Как мы убиваем двигатель своей машины летом

Начавший разрушаться подшипник какого-то вала коробки передач по расходу топлива не определишь, хотя и он немного повышает потери на трение. Зато любые пробуксовки фрикционных пар могут чувствительно повысить расход топлива. Причем это будет относится и к обычному сцеплению механической коробки, и к ремню вариатора, и к фрикционам гидромеханической коробки. Правда, ездить с такими дефектами удастся совсем недолго, а потом вы будете оплачивать топливо и прочие расходы эвакуатора.

Еще чувствуется, когда подгорело сцепление, замыкающее гидротрансформатор вариатора или автомата, ведь в режиме постоянной пробуксовки и расход топлива, и температура жидкости растут.

Перерасход топлива вызывает слишком густое масло в агрегатах трансмиссии. Также причиной может стать не отключенный передний мост, если он имеет «жесткую» схему подключения.

Ходовая часть и рулевое управление

Материалы по теме

3 неисправности, которые губят мотор: не пропустите тревожные сигналы!

Нарушение углов установки колес — как передних, так и задних — обязательно повышает расход топлива, поскольку возрастает сопротивление качению. Рецепт очевиден: проверить и подкорректировать.

Давление в шинах должно быть в норме — это очевидно. Приспущенные шины — дыра в бюджете. Недаром многие производители перед поездкой по автобанам рекомендуют немного поднять давление в шинах.

Тормозная система

Стояночная и рабочая тормозные системы легко могут поднять расход топлива хоть вдвое. Подклинивающие тормозные цилиндры, не полностью разблокированный стояночный тормоз — всё это необходимо проверить и, при необходимости, отрегулировать.

Недостаточное давление в шине быстро разрушает ее и увеличивает расход топлива.

Недостаточное давление в шине быстро разрушает ее и увеличивает расход топлива.

Перегруз и аэродинамика

Перегруженный автомобиль расходует больше топлива: это очевидно. Энергия тратится как на разгон повышенной массы, так и на сопротивление качению перегруженных шин.

Ухудшение аэродинамики — это, конечно, про багажник на крыше, всякие дефлекторы и брызговики. В космосе это не имело бы значения, а вот при наличии атмосферы сопротивление растет примерно пропорционально квадрату скорости автомобиля.

Перегруз автомобиля и значительное ухудшение аэродинамики мгновенно сказываются на расходе топлива.

Перегруз автомобиля и значительное ухудшение аэродинамики мгновенно сказываются на расходе топлива.

Что еще?

  • Псевдоспортивная манера вождения автомобиля способна повысить расход топлива до любой величины. Многим кажется, что у педалей вообще нет промежуточных положений: либо нажата полностью, либо не нажата вовсе. Вместо того, чтобы двигаться в потоке, такие водители практикуют резкие старты от светофора и обгоны в режиме «газ в пол». Про низкий расход при этом следует забыть — диагностировать нужно только себя самого.
  • Непрогретый мотор, равно как и коробка передач, даже теоретически не может работать экономично. Режим коротких поездок на холодном моторе часто называют режимом городского врача или докторским циклом. Бензин льется рекой на непрогретый нейтрализатор — так система управления пытается возможно быстрее включить его в работу. Какая уж тут экономия…
  • Некачественное топливо. Редко, но бывает — особенно там, где водителя привлекли низкие цены. Возросший расход сразу не почувствовать, но если мотор вдруг повел себя как-то непривычно, будьте уверены: его аппетит уже вырос. Диагностика простая: попробуйте доковылять до заведомо хорошей АЗС и залить качественное топливо. Если поможет — ваше счастье.
ДРОВА ВМЕСТО БЕНЗИНА

В начале 40‑х годов из Москвы в Благовещенск (Амурская область) отправился газогенераторный грузовик ГАЗ‑42. «Самовар» (так шутливо называли газогенераторные машины) за 55 ходовых дней преодолел 8400 км, использовав 23 кубометра чурок из березы, дуба, ели, сосны, лиственницы, а в безлесной местности пилили на чурки старые шпалы.

  • Подробная инструкция по снижению расхода топлива тут.
  • «За рулем» можно читать и в Телеграм.

Как снизить расход топлива — эксперты объясняют

Для этого нужно прежде всего разобраться с тем, почему двигатель стал поджирать бензин.

Как снизить расход топлива — эксперты объясняют

Улучшение расхода топлива благодаря современным конструкциям

автомобильная промышленность, Автомобильные аккумуляторные батареи 18 марта 2022

Вести: Расход топлива – важный вопрос не только с точки зрения затрат на эксплуатацию автомобиля, но и с точки зрения воздействия на окружающую среду.  Что такое средний расход топлива для автомобиля, как его рассчитать и как его улучшить? Количество топлива, сжигаемого на 100 километров, зависит от многих факторов – некоторые из них напрямую зависят от водителя, другие являются результатом конструкции автомобиля.

Что определяет расход топлива автомобиля?

В зависимости от автомобиля средний расход топлива может варьироваться от примерно пяти до более десятка литров на 100 километров. Различия между отдельными автомобилями обусловлены, прежде всего, объемом двигателя, техническим состоянием и конструкцией, а также стилем вождения водителя. Стоит помнить, что расходы на топливо составляют значительную часть расходов на эксплуатацию автомобиля. Более того, автомобили ответственны примерно за 15% выбросов углекислого газа в Европейском Союзе, а при сгорании одного литра топлива образуется от 2,35 кг (бензин) до 2,6 кг (дизель) CO2. Такое влияние на загрязнение окружающей среды обуславливает необходимость знаний для каждого водителя о том, от чего зависит расход топлива и как его можно сократить.

Размер и мощность двигателя, а также технология, используемая в приводе, оказывают значительное влияние на расход топлива. Если вы хотите снизить расход топлива, стоит покупать новые модели, которые часто оснащены дополнительными решениями, снижающими выбросы вредных веществ. Также важно обращать внимание на качество топлива, а также на техническое состояние автомобиля, чистоту и эффективность отдельных компонентов.

Стиль вождения и расход топлива

Водитель также оказывает большое влияние на расход топлива – особенно важно вести машину плавно, то есть избегать резких ускорений и торможений. Правильное использование передач – еще один фактор, который может снизить расход топлива. Также полезно использовать торможение двигателем и следить за оборотами двигателя, чтобы иметь возможность как можно быстрее переключиться на более высокую передачу. Вам также следует научиться рассчитывать расход топлива.

Как рассчитать расход топлива?

В поисках ответа на вопрос «каков расход топлива моего автомобиля?» следует ознакомиться с отчетами о сгорании топлива или воспользоваться калькулятором расхода топлива. Расчеты очень просты – достаточно разделить объем использованного топлива на количество пройденных километров, а затем умножить результат на 100. В Интернете существует множество калькуляторов, облегчающих эту задачу. Кроме того, подавляющее большинство автомобилей способны отображать текущий расход топлива – можно также проверить расход топлива на заданном расстоянии.

Бортовой компьютер автомобиля постоянно отслеживает состояние многих датчиков, в том числе и тех, что расположены на топливных форсунках. В сочетании с информацией о давлении топлива и других параметрах работы двигателя автомобиль способен дать оценку расхода топлива в данный момент.

Снижение расхода топлива на этапе проектирования автомобиля

Современный дизайн позволяет значительно снизить расход топлива. Большинство деталей в автомобилях изготавливаются из пластика, что значительно снижает их вес, а значит – и вес всего автомобиля. По этой причине стоит делать ставку на материалы, которые в первую очередь характеризуются легкостью, обеспечивая при этом соответствующие параметры в плане прочности и формуемости отдельных компонентов. Правильный выбор материалов может снизить средний расход топлива до нескольких десятков процентов.

Дизайн транспортных средств и энергоэффективность электромобилей

Оптимизация энергопотребления важна и для электромобилей. Хотя сам принцип их работы позволяет значительно снизить воздействие на окружающую среду, следует учитывать, что электроэнергия для питания двигателя часто поступает из неустойчивых источников. Кроме того, интенсивное использование аккумулятора сокращает срок его службы, вынуждая к более ранней замене батареи и увеличивая спрос на производство таких компонентов, которые также могут производить различные вредные вещества. По этой причине водители и производители электромобилей также должны уделять особое внимание энергоэффективности – как с точки зрения дизайна, так и стиля вождения.

См. также: Лучшие примеры и методология КСО в Knauf Automotive

Использование современных материалов, таких как вспененный полипропилен (EPP), дает такие же преимущества, как и в случае с двигателями внутреннего сгорания. Самым большим преимуществом является снижение общего веса автомобиля – фактор, который напрямую связан с потреблением энергии. Компоненты из EPP также имеют множество других преимуществ, например, обеспечение надлежащей защиты и изоляции литий-ионных батарей. Прочность материала и отличные теплоизоляционные свойства делают его идеальным для такого применения, а также для применения в производстве других деталей электромобилей.

См. также: Экологичные пластики – инновационные материалы для автомобильного сектора

Knauf Industries – решения для экологичных автомобилей

Компания Knauf Industries является одним из лидеров в производстве легких автомобильных деталей из пластмасс, в частности вспененного полипропилена (EPP) и пенополистирола (EPS). Благодаря использованию этих материалов продукция Кнауф позволяет значительно снизить вес автомобиля, что приводит к уменьшению расхода топлива. Гибкость материалов также позволяет создавать индивидуальные решения для изготовления компонентов, идеально подходящих к конкретной модели. Производственные процессы Knauf для легких компонентов из EPP включают использование технологии литья под давлением.

См. также: Автомобильные детали производства Knauf Automotive

Сколько весит галлон газа?

Хранение аварийного запаса бензина на лодке является очень хорошей мерой безопасности. Недостаток бензина — очень распространенное явление в лодке, поэтому убедиться, что у вас достаточно бензина, чтобы добраться до гавани, — очень разумная идея.

Вам нужно знать, какой вес вы добавляете к своей лодке, чтобы вы могли определить, сколько резервов нужно держать на борту. Давайте разберем различные расчеты веса для этанола и газа без этанола.

Чтобы помочь вам определить максимальный вес, который может безопасно выдержать ваша лодка, важно начать с основ.

Сколько весит газ?

Являетесь ли вы опытным яхтсменом или абсолютным новичком — это вопрос, на который должен ответить каждый.

Согласно справочнику по науке и технологиям , , галлон газа США весит 6,1 фунта, а британский галлон весит 7,2 фунта.

Чтобы дать вам более четкое представление о том, сколько это стоит, вот несколько вещей, о которых следует помнить:

1. Галлон газа легче галлона воды.

По сравнению с газом вода весит 8,4 фунта. Это разница более чем в 2 фунта, что объясняет, почему газ всплывает на поверхность воды. Вот почему лить воду на горящий бензин не очень хорошо.

2. Дизель намного тяжелее бензина.

Галлон дизельного топлива весит примерно 7,1 фунта. Эта разница в 1 фунт объясняется более крупными молекулами и более высокой плотностью. Его температура вспышки и температура самовоспламенения также значительно выше.

Большинство лодок длиной более 45 футов обычно работают исключительно на дизельном топливе, поэтому их двигатели обычно больше и тяжелее.

Чтобы вам было еще проще измерять, представьте несколько простых иллюстраций:

  • Галлон бензина тяжелее двухлитровой бутылки газировки
  • Галлон бензина легче двух учебников среднего размера
  • Галлон газа в два раза больше веса тостера с двумя ломтиками
  • Галлон газа равен весу маленького шара для боулинга

Британский имперский вес и вес в галлонах США: в чем разница?

Простой ответ: Имперский галлон тяжелее галлона США.

  • 1 британский галлон газа эквивалентен 7,2 фунта (3,2659 литра)
  • 1 американский галлон газа эквивалентен 6,1 фунта (2,7669 литра) или на 20% больше по объему, чем галлон США.

    Изменяется ли вес газа в зависимости от погоды?

    Да, но только до ОЧЕНЬ небольшого количества – максимум менее фунта на галлон.

    Вес газа зависит от многих факторов, в частности от влажности, температуры и погодных условий.

    Общее правило состоит в том, что в более жаркие дни бензин имеет тенденцию весить больше, потому что его объем увеличивается, что также известно как тепловое расширение. Судовое дизельное топливо, в частности, очень чувствительно к изменению температуры из-за его более высокой плотности.

    Поскольку колебания веса очень минимальны, это не сильно повлияет на общую грузоподъемность вашей лодки. Но если вы хотите заправиться дополнительным газом, в жаркие летние дни лучше брать с собой легкие вещи.

    Почему этот номер важен?

    Определение веса газа на галлон — один из самых простых способов оценки расхода топлива.

    «Сжигание топлива» — это расход топлива вашей лодкой — мера того, сколько топлива ваша лодка использует на морскую милю. Это, в свою очередь, определит, сколько запасных галлонов вам понадобится для одного рейса.

    _Boating Magazin_e придумал простую для вычислений формулу, которая измеряет эффективность использования топлива в фунтах топлива, израсходованного на лошадиную силу, развиваемую в час.

    Принимая это во внимание, понтонные и окунеобразные лодки обычно потребляют около 4–5 галлонов в час. Это означает, что для 5-часового рейса вам потребуется около 25 галлонов бензина. Для лодок с длиной корпуса более 20 футов, таких как катера с центральной консолью, средний расход топлива составляет 5–6 галлонов в час.

    Эти числа основаны на крейсерской скорости. Для прогулочного катера среднего размера эта скорость часто меньше 20 узлов. Расход топлива различается в зависимости от многих факторов, таких как управляемость и плохие погодные условия.

    Должны ли вы брать с собой дополнительный бензин во время каждой морской прогулки?

    Дело в том, что вам не нужно брать с собой дополнительный бензин, если в этом нет крайней необходимости.

    У большинства лодок есть топливный бак, в котором достаточно топлива, чтобы продержаться на воде целый день. И важно помнить, что газ чрезвычайно легко воспламеняется, поэтому слишком большое его количество на борту увеличивает риск возгорания на лодке.

    Но если вы планируете провести на воде больше нескольких дней, вам понадобится дополнительное топливо, чтобы завершить путешествие.

    В таких случаях следует помнить несколько вещей:

    1. Безопаснее всего брать с собой на борт одну газовую канистру на 6 галлонов.

    Его легко передвигать, и он не слишком сильно изменит вес вашей лодки. Этого достаточно, чтобы пробежать больше часа на крейсерской скорости.

    2. Используйте подходящие канистры, специально предназначенные для хранения топлива.

    Сюда входят баки, одобренные Береговой охраной США, и переносные/вспомогательные топливные баки. Использование пластиковых и других контейнеров может привести к вытеканию топлива.

    3. Хранить в проветриваемом помещении, но вдали от прямых солнечных лучей.

    Это связано с тем, что газ имеет тенденцию расширяться и повышать давление при воздействии тепла.

    4. Дополнительные банки должны быть надежно закреплены.

    Это можно сделать с помощью ремней и длинных креплений, чтобы они не падали и не протекали по всей лодке.

    5. При необходимости повторной заправки делайте это на открытом месте и вдали от любых источников возгорания.

    Вместимость лодки и опасность перегрузки

    Прежде чем решить, брать ли на борт дополнительную канистру с бензином, важно определить максимальную вместимость вашей лодки.

    Законы большинства штатов о судоходстве требуют, чтобы табличка грузоподъемности размещалась на прогулочных судах, особенно на лодках менее 20 футов.

    На этой табличке указана грузоподъемность лодки с точки зрения количества человек и фунтов, которые лодка может безопасно вместить. Но для целей дополнительного хранения топлива наиболее важным числом является количество фунтов.

    Перегрузка лодки большим количеством дополнительных канистр с бензином может привести к затоплению лодки. И даже если вы находитесь в пределах допустимого веса, банки должны храниться и распределяться равномерно, чтобы лодка оставалась устойчивой.

    Это важный фактор, который следует учитывать, особенно в плохую или переменчивую погоду, когда перегруженная лодка с большей вероятностью будет затоплена или даже перевернется.

    Итог

    Все успешные морские прогулки начинаются с правильного планирования и подготовки.

    Перед тем, как сесть на лодку, важно понять и правильно оценить расход топлива вашей лодки, расход бензина и максимальную вместимость — все это можно легко определить, если вы знаете, сколько весит галлон бензина.

    Сколько весит галлон газа? – Rx Mechanic

    Недостаток бензина – обычное дело для некоторых автовладельцев, поэтому необходимо держать немного бензина в резерве. Представьте себе, что бензин закончился в середине пути? Это будет неприятное и рискованное событие.

    Очень важно знать, сколько бензина требуется для работы автомобиля. Это поможет вам обуздать любой случай нехватки газа. Кроме того, большинство людей как бы размышляют о весе газа; поэтому они продолжают спрашивать: «Сколько весит галлон бензина?»

    Достаточная информация о бензине, его весе в зависимости от количества, а также о разнице между весом газа и других жидких веществ, таких как вода и дизельное топливо, представлены ниже в этой статье.

    Сколько весит газ?

    Сколько весит газ? Это фундаментальный вопрос, на который все владельцы автомобилей, особенно новички, должны ответить как часть основной информации, необходимой им для поддержания идеальной работы своего автомобиля.

    Вес газа (в фунтах) зависит от контейнера, используемого для его хранения. Справочник по науке и технологиям утверждает, что галлон бензина в США весит около 6,1 фунта. Это сильно отличается от веса галлона дизельного топлива, воды и других жидких веществ.

    Важно отметить следующие основные детали о бензине, дизельном топливе и воде, поскольку эти три вещества различаются по плотности, содержанию молекул и т. д. Таким образом, важно отметить это;

    • Бензин легче воды. Следовательно, один галлон бензина легче галлона воды.
    • Дизель тяжелее бензина. Разница в весе обусловлена ​​их плотностью, а также количеством молекул.
    • Один галлон бензина тяжелее, чем одна бутылка газировки (2-литровая бутылка).
    • Галлон бензина в два раза тяжелее тостера с двумя ломтиками.

    Сколько весит полный бак?

    Вес полного бака бензина зависит от размера автомобиля или лодки, в зависимости от обстоятельств. Например, автомобили меньшего размера поставляются с бензобаками, вмещающими около 12 галлонов бензина; в то время как более крупные автомобили поставляются с большим бензобаком, который может вмещать до 15–16 галлонов газа.

    Между тем, большинство емкостей для гриля имеют совершенно другой вес. Они весят эквивалент 17 фунтов, когда они пусты, и могут вместить до 20 фунтов бензина.

    Сколько весит галлон газа?

    Если вы задаетесь вопросом: «Сколько весит галлон газа в килограммах или фунтах?» вес галлона газа в США составляет примерно 6,1 фунта, согласно отчету Справочника по науке и технологиям. Он также эквивалентен 2,767 кг.

    Это относительно меньше, чем вес одного галлона дизельного топлива и одного галлона воды из-за их различной плотности и молекул.

    Изменяется ли вес бензина, когда становится холоднее или теплее?

    Конечно, вес газа обычно изменяется при изменении температуры, влажности и общих погодных условий. Однако разница в весе при любом из этих условий относительно невелика.

    Изменение веса газа редко превышает 1 фунт, так как в большинстве случаев оно обычно меньше. Согласно закону Шарля (закону Гей-Люссака) важно отметить, что температура и объем газа находятся в прямо пропорциональной зависимости.

    Закон гласит, что «давление данной массы газа прямо зависит от абсолютной температуры газа, когда объем поддерживается постоянным». Говоря более упрощенно, с повышением температуры бензина вместе с ним растет и давление, и наоборот.

    Различные типы бензина

    На большинстве заправочных станций в США продается 3 различных типа бензина. Разница определяется на основе их отдельного октанового числа. В сравнительной таблице ниже категорически указывается разница между этими тремя типами бензина в США

      Обычный газ Среднесортный газ Премиум газ
    Октановое число 87 Октан Октановое число 88 – 90 Октановое число 91 – 94
    Тип транспортного средства Большинство обычных автомобилей. Специальные автомобили предназначены для работы на высокооктановом газе. Суперавтомобили, разработанные с чрезвычайно высокими характеристиками, требуют бензина с очень высоким октановым числом.

    Обычный газ

    Как следует из названия, обычный газ является наиболее распространенным видом топлива, используемым в большинстве автомобилей. На самом деле, большинство производителей автомобилей рекомендуют автовладельцам использовать обычный газ.

    Этот тип газа является оптимальным стандартом для большинства автомобилей и дешевле по сравнению с другими типами газа. Он поставляется с октановым числом около 87.

    Газ среднего качества

    Газ среднего качества называется специальным газом. Оно подходит для определенных типов автомобилей и моделей, предназначенных для работы на газе с более высоким октановым числом.

    Например, некоторые внедорожники рассчитаны на использование бензина среднего качества с более высоким октановым числом. Важно отметить, что более высокое октановое число соответствует более высокой производительности двигателя. Он поставляется с около 88 — 9с октановым числом 0, и он относительно дороже обычного бензина.

    Газ премиум-класса

    Бензин премиум-класса — это тип газа с самым высоким октановым числом, продаваемый на заправочных станциях США. Этот тип газа обычно используется в транспортных средствах. Однако некоторые высокопроизводительные автомобили предназначены для работы на бензине премиум-класса с очень высоким октановым числом.

    Бензин премиум-класса имеет октановое число примерно 91–94 и стоит больше, чем бензин среднего и обычного качества. Их также называют «высокооктановым газом». Некоторые люди также называют его «супергазом», потому что он используется в сверхмощных двигателях транспортных средств.

    Часто задаваемые вопросы

    В: Сколько весит галлон неэтилированного бензина?

    Если вы спрашиваете: «Сколько весит галлон неэтилированного бензина?» Американский галлон неэтилированного бензина весит эквивалент 6,1 фунта. Это не так тяжело, как имперский галлон бензина, который весит примерно 7,2 фунта.

    Вес дополнительно разделен на 2,7669 литров (6,1 фунта) для галлона неэтилированного бензина и 3,2659 (7,2 фунта) литров для имперского галлона бензина.

    Если вы также задаетесь вопросом: «Сколько стоит галлон бензина?» Ориентировочная стоимость по средней цене на газ по стране составляет 3,416 доллара.

    В: Сколько весят 5 галлонов бензина?

    Если галлон неэтилированного бензина весит 6,1 фунта (2,7669 литра), то 5 галлонов газа будут весить соответственно 30,5 фунта (13,8345 литра). Итак, если вы задаетесь вопросом: «Сколько весит 5 галлонов бензина?» простая арифметика выше — идеальный ответ.

    Количество газа или количество галлонов напрямую влияет на вес газа. Планируя поездку на лодке или автомобиле, важно учитывать, сколько бензина потребляет двигатель вашей лодки или автомобиля в конкретной поездке.

    Также важно знать расход топлива в двигателе, а также объем бака. Все эти факторы позволят вам рассчитать достаточное количество для каждой поездки.

    В: Сколько весят 100 галлонов газа?

    Сто (100) галлонов газа весят эквивалент 700 фунтов. Маленькие автомобили с небольшой емкостью топливного бака вмещают всего до 12 галлонов бензина; в то время как другие могут вмещать до 15 галлонов.

    Между тем, бензобак лодки может вместить до 30 галлонов бензина. Если вы спросите: «Сколько весят 30 галлонов бензина?» Это где-то 183-189фунты.

    Однако размер лодки тоже имеет значение. В то время как бензобак меньшей лодки может вмещать около 5 галлонов, большая яхта сможет вместить до 250 000 галлонов бензина за раз.

    В: Что тяжелее: вода или бензин?

    Если вы думаете о весе бензина по сравнению с водой, бензин и вода являются жидкими веществами; однако их вес различается. Вода обладает более высокой плотностью по сравнению с бензином. Вот почему бензин может плавать на воде.

    В то время как галлон бензина весит эквивалент 6,1 фунта, один галлон воды весит примерно 8,4 фунта. Следовательно, вода тяжелее бензина из-за своей плотности, а также молекулярного состава.

    В: Является ли топливо для реактивных двигателей тяжелее бензина?

    Топливо для реактивных двигателей немного тяжелее бензина. В то время как бензин содержит углеводороды с числом атомов углерода от 7 до 11, топливо для реактивных двигателей состоит из углеводородов с числом атомов углерода от 12 до 15, что делает его тяжелее бензина.

    Топливо для реактивных двигателей несколько ближе к керосину, чем к газу. Также важно отметить, что реактивное топливо поставляется с впрыскиваемыми добавками, которые образуют смесь. Это содержимое делает его тяжелее бензина во всех аспектах.

    В: Сколько весит галлон бензина с октановым числом 93?

    Галлон бензина с октановым числом 93 весит эквивалент 6–6,1 фунта. Вес такой же, как у бензина с октановым числом 91. Бензин с октановым числом 93 довольно дорогой по сравнению с обычным газом.

    9Бензин с октановым числом 1–94 относится к категории бензина премиум-класса и кажется более дорогим, чем бензин среднего качества (с октановым числом 88–90) и обычный бензин (с октановым числом 87).

    В: Бензин весит так же, как дизель?

    Газ никоим образом не весит столько же, сколько дизель. Для тех, кто спрашивает: «Сколько весит галлон дизельного топлива?» Один галлон дизельного топлива весит эквивалент 7,1 фунта. Между тем, галлон бензина весит около 6,1 фунта, что делает дизель на 1 фунт тяжелее бензина.

    Разница между весом газа и дизельного топлива обусловлена ​​их различной плотностью и молекулами. Дизель имеет более высокую плотность и более крупные молекулы, чем бензин. Температуры самовоспламенения и температуры вспышки дизельного топлива также выше, чем у бензина.

    Final Words

    Планирование путешествия очень важно для определения того, насколько успешным оно закончится. Поэтому обязательно проверьте расход бензина вашего автомобиля, максимальную грузоподъемность, а также уровень содержания газа.

    Было бы легко определить, сколько вам потребуется для поездки, если бы вы могли конкретно ответить на этот фундаментальный вопрос: сколько весит галлон бензина?

    Как только вы ответите на этот вопрос, процесс подготовки станет довольно простым. Между тем, на этот вопрос для вас уже дан ответ в этой статье. Итак, вперед, планируйте свое путешествие.

    Подробнее:

    • Как очистить газовый баллончик?
    • Что произойдет, если положить сахар в бензобак?
    • Что произойдет, если залить бензин в дизельный двигатель?

    Преобразование объема бензина в вес

    Введите объем, выберите единицу объема и укажите материал или вещество для поиска. Используйте * в качестве подстановочного знака для частичного совпадения или заключите строку поиска в двойные кавычки для точного совпадения. Вес выбранного предмета рассчитывается с использованием его плотности и введенного объема.

    Объем:дюймы:кубический сантиметркубический футкубический дюймкубический метрИмперский галлонлитрметрический стаканметрическая столовая ложкаметрическая чайная ложкамиллилитрнефтяной баррельСША стаканжидкая унция СШАгаллон СШАПинта СШАКварта СШАстоловая ложка СШАЧайная ложка СШАсм. больше единиц

    Выберите состав:бензин
    точность:0123456789V2W | W2V | Плотность | Price

    show all units

    Weight of 1 cubic centimeter of petrol
    carat 3.74 ounce 0.03
    gram 0.75 pound 0
    kilogram 0 тонна 7,49 × 10 -7
    milligram 748.9  

    show all units

    The entered

    volume of petrol in various units of volume 8 британских галлонов 208 US cup9208 0.0222299
    centimeter³ 1 milliliter 1
    фут³ 3,53 × 10 -5 баррель нефти 6,29 × 10 -6
    0
    inch³ 0. 06 US fluid ounce 0.03
    liter 0 US gallon 0
    meter³ 1 × 10 -6 US PINT 0
    Metric Cup 0 US Quart 0
    SATLPON 0.022229
    TABLEPPON 0,02222 2
    . 0.07
    metric teaspoon 0.2 US teaspoon 0.2

    • About petrol
    • 1 cubic meter of petrol weighs 748.9 kilograms [kg]
    • 1 cubic foot of petrol весит 46,7523 фунта [lbs]
    • Бензин весит 0,7489 грамм на кубический сантиметр или 748,9 килограмм на кубический метр , т.е. плотность бензина .9 равна 748кг/м³. В имперской или американской системе измерения плотность равна 46,7523 фунтов на кубический фут [фунт/фут³] или 0,43289 унций на кубический дюйм [унций/дюйм³].
    • Также известен как: бензин.
    • Закладки :  [  вес к объему  | объем к весу  | цена | плотность]
    • Об этой странице:  Вес бензина
    • Например, подсчитайте, сколько унций, фунтов, миллиграммов, граммов, килограммов или тонн выбранного вещества в литре, галлоне, жидкой унции, кубическом сантиметре или в кубическом дюйме. На этой странице вычисляется вес вещества в расчете на заданный объем и дается ответ на вопрос: Сколько весит вещество в расчете на объем.
    • Ссылка (ID: 2293)
    • 1. Центр данных по альтернативным видам топлива; Энергоэффективность и возобновляемые источники энергии; Министерство энергетики США; Форрестол Билдинг; проспект Независимости, 1000, ЮЗ; Вашингтон, округ Колумбия, 20585. Последний доступ: 29 августа 2020 г. (www.afdc.energy.gov).
    Пищевые продукты, питательные вещества и калории

    ПОПКОРНОПОЛИС, ЗЕБРА, КАРАМЕЛЬНАЯ КУКУРУЗА С ПОЛОСКАМИ С ШОКОЛАДНЫМ ВКУСОМ, UPC: 815196010401 содержат 481 калорию цена за 100 грамм (≈3,53 унции) 9 [0003

    268007 пищевые продукты, содержащие углеводов, по разнице . Список этих продуктов, начиная с самого высокого содержания углеводов по разнице и с самым низким содержанием углеводов по разнице

    Гравий, вещества и масла

    CaribSea, Marine, Aragonite, Flamingo Reef весит 1 361,6 кг/м³ (85.00191 фунт/фут³) с удельным весом 1,3616 по отношению к чистой воде. Подсчитайте, сколько этого гравия требуется для достижения определенной глубины в цилиндрическом, четвертьцилиндрическом или прямоугольном аквариуме или пруду [вес к объему | объем к весу | цена ]

    Хлористый углерод, газ [COCl 2 ] весит 4,531 кг/м³ (0,00261908 унций/дюйм³) [вес к объему | объем к весу | цена | моль к объему и весу | масса и молярная концентрация | плотность ]

    Преобразование объема в вес, веса в объем и стоимости для Хладагент R-508B, жидкий (R508B) с температурой в диапазоне от -106,67°C (-160,006°F) до -6,65°C (20,03° F)

    Вес и размеры

    Фунт-сила-дюйм (фунт-сила-дюйм) — это единица измерения крутящего момента, не входящая в SI (не System International).

     Энергия – это способность выполнять работу, и она бывает разных форм: тепло (тепловая), световая (лучистая), движение (кинетическая), электрическая, химическая, ядерная и гравитационная.

    Таблица перевода фунтов/pt в т/м³, конвертер единиц фунт/pt в т/м³ или перевод всех единиц измерения плотности.

    Калькуляторы

    Калькулятор эллипса, уравнения, площади, вершины и окружности

    Распределение веса топлива | High Road Online CDL Training

    Будет много раз, когда ваш вес будет достаточно близок к разрешенным законом ограничениям веса, и вам придется следить за тем, сколько топлива вы везете. Вам также придется следить за тем, сколько топлива вы заливаете при заправке, и рассчитывать, когда вы сможете заправиться в следующий раз. Вес самого топлива и размещение топливных баков будут влиять на расход топлива и распределение веса при перевозке грузов, близких к установленным законом ограничениям.

    Вес топлива

    Точный вес дизельного топлива зависит от температуры и влажности, но обычно составляет около 7,25-7,5 фунтов на галлон. Поэтому безопаснее всего использовать 8 фунтов на галлон в качестве числа, которое вы будете использовать в своих расчетах, и это то, что мы собираемся использовать здесь.

    Распределение веса бортового топлива

    Вес топлива в ваших баках будет распределяться в основном на управляемую ось, а часть приходится на ведущие оси. Процент веса, который приходится на каждый комплект осей, будет варьироваться от грузовика к грузовику в зависимости от размещения ваших баков относительно ваших управляемых и ведущих осей.

    Лучший способ определить, какой процент вашего топлива пойдет на каждый комплект осей, — это взвесить грузовик непосредственно до и после заправки и выполнить простой расчет.

    Взвешивание грузовика для определения распределения веса топлива

    Чтобы точно определить, какой процент веса топлива идет на управляемую ось по сравнению с ведущими мостами, вы можете просто взвесить грузовик, затем сразу же заправить его топливом, а затем немедленно снова взвесить грузовик. чтобы увидеть, какой процент добавленного веса пошел на управляемую ось, а какой процент — на ведущие оси. Вот пример того, как делать расчеты:

    Вы взвешиваете свой грузовик перед заправкой и получаете квитанцию ​​о взвешивании. Вот веса:

    • Рулевое управление:

      8000

    • Диски:

      13 000

    • Трейлер:

      13 000

    • Валовой доход:

      34 000

    После заливки 100 галлонов топлива вы немедленно повторно взвешиваете грузовик, и вот ваш новый весовой талон:

    • Рулевое управление:

      8 700

    • Диски:

      13 100

    • Трейлер:

      13 000

    • Валовой доход:

      34 800

    Таким образом, взглянув на квитанцию ​​на весах, вы увидите, что по общему весу вы добавили 800 фунтов топлива. 700 фунтов ушло на бычки, а 100 фунтов — на приводы. Таким образом, чтобы рассчитать процент веса топлива на рулевое управление по сравнению с приводом, формула будет следующей:

    Возьмите вес, добавленный к управляемым шинам (700 фунтов), и разделите его на общий вес добавленного топлива (800 фунтов), а затем умножьте это на 100:

    700/800 * 100 = 87,5% пришлось на управляемые колеса, а на ведущие мосты пришлось 12,5 % (100 % — 87,5 % = 12,5 %). Теперь каждый раз, когда вы получаете топливо, вы будете точно знать, куда пойдет вес.

    CDL:

    Коммерческие водительские права (CDL)

    CDL требуется для управления любым из следующих транспортных средств:

    • Любая комбинация транспортных средств с номинальной полной массой (GCWR) 26 001 или более фунтов, при условии, что номинальная полная масса буксируемого транспортного средства (GVWR) превышает 10 000 фунтов.
    • Любое отдельное транспортное средство с полной разрешенной массой 26 001 фунт или более или любое такое транспортное средство, буксирующее другое, не превышающее 10 000 фунтов.
    • Любое транспортное средство, независимо от размера, предназначенное для перевозки 16 и более человек, включая водителя.
    • Любое транспортное средство, требуемое федеральными правилами, должно иметь табличку при перевозке опасных материалов.

    Какое безопасное число использовать при расчете веса топлива?

    • 7,25 фунтов на галлон
    • 6 фунтов на галлон
    • 10 фунтов на галлон
    • 8 фунтов на галлон
    Цитата из руководства CDL:

    Точный вес зависит от температуры и влажности дизельного топлива. но обычно составляет около 7,25-7,5 фунтов за галлон. Поэтому безопаснее всего использовать 8 фунтов на галлон в качестве числа, которое вы будете использовать в своих расчетах, и это то, что мы собираемся использовать здесь.

    Вы взвешиваете грузовик непосредственно перед заправкой и после нее. Основываясь на следующем, определите, какой процент веса приходится на каждый комплект осей:

    До заправки:
    рулевое управление: 10 200, приводы: 31 300, брутто: 71 700
    После заправки:
    рулевое: 11 000, приводы: 31 500, брутто: 72 700

    • 67% пошло на управляемую ось
      33 % приходится на ведущие оси
    • 70 % приходится на управляемую ось
      30% ушло на ведущие мосты
    • 75% ушло на управляемую ось
      25% пришлись на ведущие мосты
    • 80% пришлись на управляемый мост
      20% ушло на ведущие мосты
    Цитата из Руководства CDL:

    Чтобы определить процент веса топлива, добавленного к управляемому мосту, возьмите вес, добавленный к управляемому мосту, разделите его на общий добавленный вес топлива (определенный на изменение веса брутто), а затем умножьте это на 100.

    TruckingTruth’s Advice:

    Чтобы определить процент веса топлива, добавленного к управляемой оси, возьмите вес, добавленный к управляемой оси (800 фунтов), разделите его на общий добавленный вес топлива (1000 фунтов), а затем умножьте полученное значение. раз 100

    800/1000 = .8

    .8 * 100 = 80 % массы топлива на управляемые колеса

    100 % — 80 % = 20 % пошло на ведущие мосты.

    Предыдущая

    Следующая

    Куда будет распределяться масса топлива?

    • В основном на ведущие мосты, а некоторые на управляемый мост.
    • В основном на управляемую ось, часть на ведущие оси.
    • Полностью на управляемую ось
    • Полностью на ведущую ось
    Цитата из руководства CDL:

    Вес топлива в ваших баках будет распределяться в основном на управляемую ось, а часть на ведущие оси.

    Предыдущий

    Следующий

    Вы взвешиваете грузовик непосредственно перед и после заправки топливом. Основываясь на следующем, определите, какой процент веса приходится на каждый комплект осей:

    До заправки:
    рулевое управление: 11 450, приводы: 33 100, полная: 76 700 77 300

    • 73% пошло на управляемую ось
      27 % приходится на ведущие оси
    • 88 % приходится на управляемую ось
      12 % приходится на ведущие оси
    • 67 % приходится на управляемую ось
      33 % приходится на ведущие оси
    • 75 % приходится на управляемую ось
      25% ушло на ведущие мосты
    Цитата из руководства CDL:

    Чтобы определить процент массы топлива, добавленной к управляемой оси, возьмите массу, добавленную к управляемой оси, разделите ее на общую добавленную массу топлива (определяется на изменение веса брутто), а затем умножьте это на 100

    Совет TruckingTruth:

    Чтобы определить процент веса топлива, добавленного к управляемой оси, возьмите вес, добавленный к управляемой оси (400 фунтов), разделите его на общий добавленный вес топлива (600 фунтов), а затем умножьте полученное значение. умножить на 100

    400/600 = 0,67

    0,67 * 100 = 67 % массы топлива на управляемые колеса

    100 % — 67 % = 33 % пошло на ведущие мосты.

    Предыдущий

    Следующий

    Вы взвешиваете грузовик непосредственно перед и после заправки топливом. Основываясь на следующем, определите, какой процент веса приходится на каждый комплект осей:

    До заправки:
    рулевое управление: 11 100, приводы: 32 600, брутто: 71 400
    После заправки:
    рулевое управление: 11 850, приводы: 32 850, брутто: 72 400

    • 75% пошли на управляемую ось
      25% пришлись на ведущие мосты
    • 80% пришлись на управляемый мост
      20 % приходится на ведущую ось
    • 65 % приходится на управляемую ось
      35 % приходится на ведущие оси
    • 67 % приходится на управляемую ось
      33% пришлось на ведущие мосты
    Цитата из руководства CDL:

    Чтобы определить процент веса топлива, добавленного к управляемой оси, возьмите вес, добавленный к управляемой оси, разделите его на общий добавленный вес топлива (определяется изменением полной массы ), а затем умножьте это на 100.

    TruckingTruth’s Advice:

    Чтобы определить процент веса топлива, добавленного к управляемой оси, возьмите вес, добавленный к управляемой оси (750 фунтов), разделите его на общий добавленный вес топлива (1000 фунтов), а затем умножьте полученное значение. раз 100

    750/1000 = 0,75

    0,75 * 100 = 75% массы топлива на управляемые колеса

    100% — 75% = 25% ушло на ведущие мосты.

    Предыдущая

    Следующая

    Как лучше всего определить, какой процент веса топлива пойдет на каждый комплект осей?

    • Процент стандартизирован среди производителей и одинаков для всех грузовиков
    • Просто определите количество галлонов топлива, которое вмещает каждый бак
    • Взвесьте грузовик непосредственно перед и после заправки и выполните простой расчет
    • Спросите другого водителя, каковы их номера
    Цитата из руководства CDL:

    Лучший способ определить, какой процент вашего топлива пойдет на каждый комплект осей, — это взвесить грузовик непосредственно до и после заправки топливом и выполнить простой расчет

    Предыдущая

    Следующая

    Как правильно определить, какой процент топлива распределяется между управляемой и ведущей осью?

    • Взвесьте грузовик, затем немедленно заправьте его топливом, а затем сразу же повторно взвесьте грузовик, чтобы увидеть, какой процент добавленного веса приходится на управляемую ось, а какой процент приходится на ведущие оси.
    • Взвесьте грузовик сразу после заправки, затем проедьте ровно 100 миль, а затем снова взвесьте грузовик, чтобы увидеть, какой процент дополнительного веса приходится на управляемые оси.
    • Взвесьте грузовик, затем проедьте ровно 100 миль, а затем повторно взвесьте грузовик, чтобы увидеть, какой процент добавленного веса приходится на управляемую ось, а какой процент приходится на ведущие оси.
    • Взвесьте грузовик после пробега 100 миль, затем немедленно заправьте топливом, а затем снова взвесьте грузовик, чтобы увидеть, какой процент добавленного веса приходится на управляемую ось, а какой процент приходится на ведущие оси.
    Цитата из руководства CDL:

    Чтобы точно определить, какой процент веса топлива идет на управляемую ось по сравнению с ведущими мостами, вы можете просто взвесить грузовик, затем сразу же заправить его топливом, а затем немедленно повторно взвесить грузовик. чтобы увидеть, какой процент добавленного веса пошел на управляемую ось, а какой процент — на ведущие оси.

    Предыдущий

    Конечный

    Выберите один из вариантов

    [4,4,2,3,1,3,1]

    7

    Сколько весит галлон дизельного топлива?

    Раскрытие информации: Мы можем получать комиссионные за покупки, сделанные по ссылкам в этом посте.

    Транспортные средства различных размеров, от гигантоподобных коммерческих грузовиков до четырехколесных внедорожников, выигрывают от дизельного топлива. Дизельное топливо имеет разные названия по всему миру — DERV (автомобиль с дизельным двигателем) в Великобритании, дистиллят в Австралии и Solar в Индонезии, и это лишь некоторые из них — и его несколько типов и смесей широко используются в коммерческих целях. Точно так же существуют разные единицы, используемые для его измерения. В то время как большинство стран перешли на использование литров для измерения топлива в целом, некоторые места и группы энтузиастов и механиков по-прежнему предпочитают галлон старой школы. Теперь вы можете ожидать, что стоимость галлона будет постоянным числом, верно? К сожалению, это не так — или, по крайней мере, это не так просто.

    Сколько весит галлон дизельного топлива? Вес зависит от сорта топлива и используемой единицы измерения. Галлон США весит 6,82-8,0 фунтов. Сухой галлон США и имперский галлон весят 7,93–9,32 фунта и 8,19–9,61 фунта соответственно. При взвешивании дизельного топлива лучше всего использовать галлон США.

    В этой статье вы узнаете больше не только о цетановом числе и вариантах измерения, но и о других факторах, таких как температура и научные испытания качества, которые влияют на вес галлона дизельного топлива.

    Измерение дизельного топлива

    Измерение веса дизельного топлива на галлон может быть довольно сложным, поскольку существуют разные типы дизельного топлива. Не говоря уже о том, что плотность каждого вида непостоянна из-за нескольких факторов. Однако теперь легче определить это значение, особенно если у вас уже есть значения плотности (фунт-масса на кубический фут — фунт/фут3) для типа дизельного топлива, для которого вы хотите произвести расчет. Вы должны получить плотность этого конкретного топлива и разделить число на 7,48 (7,48 фунта/фут3 эквивалентно 1 фунту на галлон США), 6,23 (6,23 фунта/фут3 эквивалентно 1 фунту на британский галлон). ), или 6,43 (6,43 фунта/фут3 эквивалентны 1 фунту на сухой галлон США). Внимательно изучите калькуляторы или конвертеры, которые появятся в результатах поиска в Интернете, поскольку некоторые используют фунт на кубический фут (фунт/фут3) для расчета веса топлива в галлонах. Существует большая разница между фунтами на фут3 и фунтами на фут3.

    Почему это важно?

    Зная ответ на вопрос «сколько весит галлон дизельного топлива?» никогда не может быть подорвана. Как правило, водители должны знать свое оборудование и нагрузку на ось при погрузке или заправке топливом — это особенно важно для дальнобойщиков, которые управляют транспортными средствами, на которые распространяются ограничения по весу. Вес дизельного топлива на галлон также влияет на расчет загрузки топлива на корабле или автоцистерне. Важно обеспечить правильные характеристики цилиндрического резервуара, в котором хранится топливо, и поддерживать оптимальную дифферентовку круизного лайнера, которая помогает ему оставаться стабильным даже в неблагоприятную погоду. Другая причина заключается в том, что когда вы заполняете бак топливом, вес автомобиля увеличивается, а экономия топлива снижается, что снижает расход топлива. Для внедорожников это включает в себя уход за своим полноприводным автомобилем или квадроциклом, поскольку они не захотят, чтобы их двигатель протекал или разливался, загружая свои топливные баки сверх его емкости.

    Что влияет на вес дизельного топлива?

    • Единица измерения. В то время как различные единицы представляют плотность – г/см 3 , кг/л, кг/м3 , фунт/фут3, или порций/фут3  –   фактическое значение, которое вы получаете, близко к постоянному. Что может вызвать несоответствие в определении того, сколько весит один галлон дизельного топлива, так это то, как вы его конвертируете. Используете ли вы американские, британские или канадские единицы измерения? Вы рассчитываете галлоны США, сухие галлоны или имперские галлоны? В настоящее время используются три различных единицы измерения (значения округляются до второго знака после запятой): имперский галлон (импгал = 4,55 литра), американский галлон (usgal = 3,76 литра) и сухой галлон США (usdrygal = 4,40 литра). ). Хотя британский галлон весит больше всего, галлон США является стандартом, используемым на международном уровне.
    • Температура. Важно помнить, что объем топлива колеблется в зависимости от влажности, температуры или погодных условий. При комнатной температуре дизельное топливо составляет менее 7 фунтов/галлон. При 32 ° F это чуть более 7 фунтов. А при очень высоких температурах 100°F и выше галлон дизельного топлива будет весить около 6,8 фунтов. Из-за этой разницы измерение плотности топлива происходит при стандартных температурах 15°C или 59°F. Точно так же вязкость дизельного топлива увеличивается с понижением температуры. При температуре от -19 ° C (-2,2 ° F) до -15 ° C (5 ° F) его форма превращается в гель, который не только забивает топливопроводы и фильтры, но и делает его немного тяжелее.
    • Удельный вес и API. Это   стандарт, установленный Американским институтом нефти для определения веса нефти по отношению к воде и необходимый для преобразования объема дизельного топлива, скажем, в галлоны США. Более того, это общепринятый полевой тест для определения качества и уровня энергии дизельного топлива – чем выше API, тем ниже энергия; чем выше SG, тем выше выход энергии. Для чистого биодизеля он имеет удельный вес (S.G.) 0,88. Дизельные смеси с содержанием серы, такие как ULSD, имеют сравнительно более высокие значения, чем биодизельное топливо, что способствует снижению его веса. 2D или дизельное топливо номер 2 будет находиться в диапазоне 0,876-0,802. В целом удельный вес дизельного топлива может варьироваться от 0,82 до 0,9.0, в результате чего его вес колеблется от 6,85 до 7,5 фунтов на галлон.
    • Марка топлива.  Уровень цетанового числа дизельного топлива оценивает легкость воспламенения, мощность и экономию топлива и относится к его общему весу. Когда достигается в процессе очистки, более высокое цетановое число приводит к более высокой плотности в градусах API и более низкой плотности топлива. Это означает, что дизельное топливо в Великобритании будет иметь меньший весовой диапазон 6,95–7,04 фунта по сравнению с американским, поскольку минимальное цетановое число в Европе выше, чем в других странах. Как и в Европе, качество топлива в Канаде соответствует стандарту EN 59.0 стандартов, в результате чего дизельное топливо весит меньше.

    Различные типы дизельного топлива

    По словам инженеров и специалистов по топливу, тип смеси дизельного топлива, среди прочих факторов, может влиять на вес галлона дизельного топлива. И поскольку мы определяем точную массу дизельного топлива на галлон, очень важно понимать его различные классификации и свойства, влияющие на вес.

    Существует три различных класса дизельного топлива: 1D, 2D и 4D. Разница между ними зависит от вязкости (сопротивления жидкости течению), диапазона температур кипения, температуры застывания (самая низкая температура, при которой жидкое вещество будет течь) и цетанового числа. Я пошел дальше и вычислил их эквивалент в фунтах на галлон на основе их различной плотности (которую я получил из Engineering Toolbox) следующим образом:

    Топливо Плотность при 59°F (15°C)
    ρ –
    Вес Удельный объем
    v –
    (кг/м 3 ) (фунт/фут 3 ) (фунт/галлон США) (lb/usdrygal) (фунт/импгал) 3 /1000 кг) (фут 3 за тонну)
    Дизель 1D 875 54,6 7,30 8,49 8,76 1,14 40,4
    Дизель 2D 849 53 7,09 8,24 8,51 1,18 41,6
    Дизель 4D 959 59,9 8. 00 9,32 9.61 1,04 36,8
    EN 590 Дизель 820-845 51-53 6,82-7,08 7,93-8,24 8.19-8.51 1,18-1,22 42-43

    Дизельное топливо 1D или номер 1 химически похоже на керосин и предпочтительнее для холодных погодных условий из-за его более низкой вязкости. Этот тип дизельного топлива включает красное дизельное топливо, зимнее дизельное топливо, предназначенное для использования на бездорожье, в основном в тракторах, внедорожной технике или бытовом топливе. Он окрашен в красный цвет, чтобы отделить его от обычного дизельного топлива, поскольку красный дизель не разрешен на дорогах и не облагается налогом, как последний.

    Дизель 2D

    С другой стороны, дизель 2D или номер 2 является наиболее распространенной формой, используемой в обычных и теплых погодных условиях, и имеет более высокую вязкость, чем 1D. Иногда его смешивают с топливом 1D, чтобы получить подходящее зимнее топливо. В некоторых штатах галлон дизельного топлива 2D весит от 6,96 до 6,91 фунта при комнатной температуре от 59°F до 76°F. При более низких температурах, таких как 32 ° F, галлон дизельного топлива будет весить около 7,05 фунтов.

    Ниже приведены примеры 2D-дизеля:

    • Нефтяное дизельное топливо, также известное как фракционный дистиллят, нефтедизельное топливо или ископаемое дизельное топливо, является наиболее распространенным типом и подвергается кипячению или перегонке сырой нефти для удаления непригодных частей.
    • Синтетическое дизельное топливо состоит из углеродсодержащего материала, включая биомассу, биогаз, природный газ, уголь и многие другие, и обычно называется преобразованием биомассы в жидкость (BTL), преобразованием газа в жидкость (GTL) или преобразованием угля в жидкость. жидкость (ЦТЛ). Он также имеет парафиновый вид с почти полным отсутствием серы и очень минимальным содержанием ароматических соединений, что снижает нерегулируемые выбросы вредных газов. ДМЭ (диметиловый эфир) также относится к этому типу и также безопасен для окружающей среды.
    • Биодизель В5 состоит из 5% биодизеля и 95% обычного дизельного топлива и содержит длинную цепь жирных кислот из растительного масла или животных жиров. B5 — единственный тип биодизеля, который соответствует гарантиям производителя и регулируется стандартом ASTM D6571, хотя большинство двигателей могут работать на смесях с содержанием до 30%.
    • Зеленое дизельное топливо или гидрогенизированное масло и жиры представляют собой нечто среднее между синтетическим и биодизельным топливом. Но в отличие от биодизеля, он использует процесс очистки и гидрогенизации для превращения растительного масла и животных жиров в алканы. Многие его свойства чем-то схожи с синтетическим дизельным топливом.
    • Дизельное топливо со сверхнизким содержанием серы (ULSD) — варианты S15, S500 и S5000 использовались на автомагистралях до 2006 г. и на бездорожье до 2014 г., но в настоящее время сняты с производства из-за экологических соображений. Остается только дизельное топливо со сверхнизким содержанием серы (ULSD) S15, которое к 2014 году становится стандартом. Его более дорогой вариант, MK-1 (экологическое дизельное топливо класса 1), продается в Швеции.
    • Судовое дизельное топливо имеет свой набор вариантов. Его вид MGO (судовой газойль) эквивалентен 2D-топливу, изготовленному из дистиллята.

    Наконец, дизель 4D или номер 4 используется для стационарных агрегатов и низко-/среднеоборотных двигателей, таких как железнодорожные локомотивы и электрогенераторы, которые работают с относительно постоянной скоростью. Дизель 4D имеет более высокую плотность 59,9 фунта/фут3 или 8 фунтов на галлон, чем дизель 1D и 2D.

    Ограничения по весу – дополнительная история

    Теперь, когда вы знаете больше о вариациях галлонов, различных типах и размерах дизельного топлива, было бы полезно понять, какое значение имеет вес дизельного топлива на галлон. Большинство внедорожников отказываются от этого. Но для водителей грузовиков (и владельцев частных транспортных средств в некоторых штатах) это часть их рутины. И если они не будут осторожны при проверке веса своего топлива, дизельного топлива и полной массы автомобиля, им может быть ограничен въезд с огромными штрафами за превышение веса или они могут быть привлечены к ответственности за нарушение ограничений по весу.

    Квадроциклы и полноприводные автомобили

    Ограничения по массе в настоящее время не распространяются на квадроциклы и полноприводные автомобили, которые в основном предназначены для бездорожья. Однако выдача водительских удостоверений для частных и некоторых коммерческих транспортных средств имеет соответствующие ограничения полной массы транспортного средства (GVM). В Австралии водительское удостоверение ограничено вождением транспортных средств максимальной массой до 4500 кг (9921 фунт), называемых легковыми автомобилями. В Великобритании стандартные водительские права класса B ограничивают вес владельца лицензии до 3500 кг (7700 фунтов). В США ограничения по весу составляют 6000 и 8500 фунтов (2722 и 3856 кг) соответственно. Некоторые города ограничивают въезд транспортных средств весом более 6000 фунтов, в то время как коммерческие автомобили весом более 8500 фунтов должны быть застрахованы в соответствии с федеральными законами.

    Грузовые автомобили

    Для грузовых автомобилей, однако, Министерство транспорта США установило ограничения по весу для тех, кто работает в межгосударственных системах, специальные положения для транспорта на природном газе и электрических батареях, а также единиц для перевозки имущества в некоторых штатах с 1974 года. разрешенный вес транспортного средства составляет 80 000 фунтов, с ограничением в 20 000 фунтов на одну ось, ограничением в 34 000 фунтов на тандемную ось и ограничением в 42 000 фунтов на тройную ось. Для грузовиков с электрическим аккумулятором или природным газом разрешено превышать предельный вес на 2000 фунтов. (или на 2550 фунтов, если учитывать допуск на ВСУ), чтобы покрыть вес силовой установки автомобиля. Одноосные весы измеряют вес оси, а общий вес представляет собой вес оси, вес привода и вес груза (если есть) вместе взятые. Коммерческие транспортные средства, которые весят более 10 000 фунтов, должны будут останавливаться для проверки — это будут грузовые автомобили класса 3 и выше, пассажирские или специальные автомобили, а также автомобили для отдыха и автобусы в Пенсильвании.

    США, Великобритания и Канада

    Хотя в Великобритании ограничения по весу почти такие же, как в США, а в Канаде выше, эти страны измеряют свои ограничения по весу по-разному: британцы используют тонны, а канадцы — килограммы. Кроме того, полная масса транспортного средства определяется с использованием различных наборов весов. На некоторых весовых станциях используются одноосные весы, которые работают, когда грузовик проезжает по ним ось за осью. Другие используют универсальные весы, которые взвешивают каждую ось одновременно. Кроме того, есть весы в движении (WIM), которые, возможно, являются наиболее эффективным методом взвешивания. Водитель проезжает через датчик, после чего рассчитывается общий вес (водителю даже не нужно останавливаться).

    Знание законов вашего штата или поселка об ограничении веса, весов, используемых на ваших весовых станциях, и того, какую роль во всем этом играет вес вашего дизельного топлива, имеет решающее значение для того, чтобы уберечь вас от неприятностей. Кроме того, учет других факторов определит буфер, который у вас останется для дизельного топлива. Выясните, имеет ли ваш грузовик стандартную, разнесенную или тандемную ось. Узнайте местонахождение вашего топливного бака, если он находится под дверями кабины или койкой. Учтите, есть ли в вашем прицепе холодильная установка.

    Совет. Если вы обнаружите, что ваш вес приближается к установленному законом пределу, не забудьте не заправляться топливом непосредственно перед подходом к станции взвешивания. Заранее узнайте, где на вашем маршруте расположены станции взвешивания, и рассчитайте количество заправок. Или вы можете выгрузить часть вашего топлива в топливный бак (см. на Amazon) и попросить помощи у коллеги, чтобы нести их, если они не исчерпаны по весу.

    Часто задаваемые вопросы

    • Сколько фунтов составляет 50 галлонов дизельного топлива? Используя   галлонов США, 50 галлонов дизельного топлива будут эквивалентны 365 фунтам для дизельного топлива 1D, 354,50 фунтам для 2D и 400 фунтам для 4D при условии, что вы измерили 50 галлонов при температуре 15°C или 59°F. Я рекомендую использовать внутренний/наружной термометр Oregon Scientific EMR201 с беспроводным пультом дистанционного управления и мигающим светодиодным индикатором обледенения (см. на Amazon), чтобы проверить, взвешиваете ли вы дизельное топливо при комнатной температуре.
    • Сколько весят 60 галлонов дизельного топлива? При температуре 15°C или 59°F, 60 галлонов дизельного топлива будут эквивалентны 438 фунтам для дизельного топлива 1D, 425,40 фунтам для 2D и 480 фунтам для 4D, но могут незначительно меняться при более низких или более высоких температурах. Опять же, я рекомендую использовать галлоны США, которые являются международным стандартом.
    • Сколько весят 100 галлонов дизельного топлива? Сто галлонов дизельного топлива будут эквивалентны 730 фунтам для дизельного топлива 1D, 709 фунтам для дизельного топлива 2D и 800 фунтам для дизельного топлива 4D при температуре 15°C или 59°F и использовании галлонов США в качестве единицы измерения.
    • Можно ли округлить вес дизельного топлива на галлон до 8 фунтов для целей масштабирования? Это зависит от того, используете ли вы марку дизельного топлива или единицу измерения для расчета. Например, если вы основываете эту оценку на галлонах США, то 8 фунтов охватывают все типы дизельного топлива (от 1D до 4D). Но если вы оцениваете конкретно, скажем, дизель 2D, то 8 фунтов могут быть неточными, поскольку вес дизельного топлива соответствует 8,51 фунта с использованием британских галлонов.
    • В каких областях промышленности было бы полезно знать, сколько весит 1 галлон дизельного топлива? Водители грузовиков, судоходные линии, резервуарные хранилища, перепродавцы очищенной нефти и топлива, а также самолеты получат большую выгоду.
    • Должны ли владельцы внедорожников знать, сколько весит один галлон дизельного топлива? Да — это поможет им управлять расходом бензина своего автомобиля и своим бюджетом на топливо! Эта информация также будет полезна, когда они планируют внедорожные приключения в разные сезоны.
    • Дизельное топливо весит больше, чем газ? Да, из-за более крупных молекул и более плотного вещества. Для сравнения, дизель весит 7,09 г.фунтов на галлон США, в то время как газ (он же бензин или бензин) весит всего 6,25 фунтов на галлон США.

    Заключение. Сколько весит галлон дизельного топлива?

    Благодаря стандартам улучшения дизельного топлива и тому, что потребители стали более экономными и сосредоточенными на экономии топлива, знание того, сколько весит дизельное топливо, никогда не было таким ценным. Умение считать в уме поможет вам во многих вещах — в подготовке к поездкам на выходных, максимальном расходе топлива вашего автомобиля и отказе от огромных штрафов и штрафов за перевес.

    Теперь вы знаете, сколько весит один галлон дизельного топлива – используйте его с пользой.

    Топливо и вес — Документация по моделированию FlyByWire

    В этом разделе представлена ​​информация о конфигурации грузов A32NX и сведения о том, как использовать и ссылаться на бортовые/симуляционные функции для правильной настройки самолета для вылета.

    Существуют различия между нашей стабильной версией и версией для разработчиков.
    Пожалуйста, выберите правильную версию ниже.

    Версия для разработчиков Стабильная версия


    Общая информация о топливе и весе (нажмите, чтобы развернуть)

    Рабочий пустой вес

    Важно понимать, что термин «рабочий пустой вес» может показаться немного общим. OEW обычно рассчитывается как «пустой вес производителя + стандартные изделия + изделия оператора». Эти элементы могут быть следующими (не все включено):

    • Члены экипажа
    • Руководства
    • Еда и напитки
    • Аварийное оборудование
    • и любое оборудование/принадлежности, которые оператор считает стандартными

    При рассмотрении вашего ZFW ниже, пожалуйста, примите во внимание вышеуказанную информацию с OEW, уделяя особое внимание тому, что этот вес действительно включает летный экипаж.

    Масса нулевого топлива

    Проще говоря, нулевой вес топлива (ZFW) = OEW + полезная нагрузка.

    Полезная нагрузка определяется как пассажиры, багаж и груз.

    В ситуации, когда ваша полезная нагрузка будет примерно 14000 кг — на основе нашего нового планера: ваш ZFW будет в сумме — 56 500 кг (42 500 кг + 14 000 кг)

    Вы можете сослаться на это в отношении любого OFP, созданного вами с помощью simBrief. Для использования нашей интеграции simBrief — читайте здесь.

    Центр тяжести нулевого веса топлива

    A32NX автоматически рассчитает ZFW и ZFWCG на странице INIT B после загрузки дрона и связанной с ним полезной нагрузки в симулятор.

    Это не ваш центр тяжести для обрезки.

    • См. раздел «Вес и баланс» в руководстве по интеграции simBrief.
    • Вы можете прочитать о конфигурации INIT B здесь.

    Центр тяжести

    Допустимый диапазон центровки при взлете для A32NX составляет 16-40%. ЦТ уравновешивается в процессе загрузки пассажиров.

    Балансировка самолета для взлета обычно необязательна и технически не требуется. Пожалуйста, обратитесь к нашему контрольному списку FBW, чтобы установить соответствующую отделку, если хотите.

    В идеале все, что меньше 25 % CG, считается нагрузкой FWD, а все, что больше 25 %, считается нагрузкой AFT. При выборе между любой из конфигураций ЦТ (назад/вперед) обратите внимание на приведенную ниже информацию.

    Примечания по различным конфигурациям компьютерной графики

    Есть несколько аргументов, которые стоит учитывать, когда дело доходит до выбора ЦТ сзади или ЦТ спереди. Как правило, задний ЦТ обеспечивает лучшие летно-технические характеристики самолета (более низкие скорость сваливания, лобовое сопротивление и угол атаки при заданном коэффициенте подъемной силы), но в целом ухудшает устойчивость по тангажу.

    Для небольших самолетов, таких как A320neo, большинство эксплуатантов предпочли бы заднюю загрузку центра тяжести для экономии топлива, учитывая срок службы парка и то, насколько легко можно получить преимущества.

    Конверты компьютерной графики для самолетов (щелкните, чтобы развернуть)

    Конверты компьютерной графики для самолетов

    Наша динамическая диаграмма CG и полезной нагрузки предоставляет соответствующую визуальную информацию во время планирования и выполнения загрузки пассажиров/груза. Полностью понимая, как продольная огибающая ЦТ, воздействующая на самолет, представляет собой довольно длинную извилистую концепцию. Ниже приведено отличное руководство по пониманию диаграммы и тому, на что указывает предоставленная информация.

    `


    Конфигурация A32NX

    Убедитесь, что у вас есть наш последний профиль simBrief

    Окно MSFS Fuel & Weights на панели инструментов

    Мы заблокировали элементы пользовательского интерфейса в окне MSFS Fuel and Weights. Однако ползунки в окне MSFS подвижны, но в течение 1-3 секунд уровни топлива и полезной нагрузки должны вернуться к исходным значениям.

    ОБРАТИТЕ ВНИМАНИЕ мы изменили загрузку топлива и полезной нагрузки.

    • Топливо: теперь через EFB.
    • Payload (Dev+Exp): выполняется на вкладке Payload в EFB. См. раздел «Веса и балансировка»
    • .
    • Полезная нагрузка (стабильная): выполнено через W&B в MCDU

    Гири

    • OEW (пустой вес): 42500 кг / 93697 фунтов
      • Также называется DOW (сухой рабочий вес), который можно увидеть в других форматах simBrief OFP, таких как EZY
    • MZFW (макс. вес без топлива): 64300 кг / 141757 фунтов
    • MTOW (максимальная взлетная масса): 79000 кг / 174165 фунтов
    • MLW (максимальная посадочная масса): 67400 кг / 148591 фунтов

    Полная масса FMS (FMS GW)

    Полная масса рассчитывается как:

    Количество топлива (непосредственно из топливной системы тренажера) + ввод ZFW в MCDU.

    Значение полной массы (GW) в ECAM появится только при соблюдении определенных условий:

    • На странице (INIT FUEL PRED) указано значение ZFW/ZFWCG. Напоминание: После запуска двигателей INIT FUEL PRED переходит на страницу FUEL PRED.
    • По крайней мере, один двигатель работает.

    Топливо

    • Максимальный запас топлива: 41989 фунтов/19,046 кг
    • Топливные баки: 5
      • 2 внешних бака: 1528 фунтов/693 кг каждый
      • 2 внутренних бака: 12167 фунтов/5519 кг каждый
      • 1 центральный бак: 14599 фунтов/6622 кг

    Пассажиры

    • Максимальная пассажировместимость: 18096 кг (174 места * 104 кг)
    • Вес пассажира: 104 (в килограммах)
      • 84 кг для пассажира (включая одежду и ручную кладь)
      • 20 кг для зарегистрированного багажа
      • Эти значения по умолчанию можно перезаписать, изменив используемый планер simBrief. См. Изменение весов по умолчанию с помощью simBrief.
    • Пассажирские салоны: 4
      • ЭКОНОМИЧЕСКИЕ РЯДЫ 1-6 (места: 36 макс: 6670 фунтов/3024 кг)
      • ЭКОНОМИЧЕСКИЕ РЯДЫ 7–13 (места: 42 макс. : 7780 фунтов/3530 кг)
      • ЭКОНОМИЧЕСКИЕ РЯДЫ 14–21 (места: 48 макс.: 8880 фунтов/4032 кг)
      • ЭКОНОМИЧЕСКИЕ РЯДЫ 22–29 (места: 48 макс.: 8880 фунтов/4032 кг)

    Груз

    • Максимальная грузоподъемность: 20800 фунтов/9435 кг
    • Грузовые отсеки: 4
      • ПЕРЕДНИЙ БАГАЖ/КОНТЕЙНЕР (макс.: 7500 фунтов/3402 кг)
      • КОРМОВОЙ КОНТЕЙНЕР (макс.: 5350 фунтов/2426 кг)
      • ЗАДНИЙ БАГАЖ (макс.: 4650 фунтов/2110 кг)
      • ЗАДНЯЯ НАГРУЗКА/НАГРУЗКА (макс.: 3300 фунтов/1497 кг)

    Изменение веса по умолчанию с помощью simBrief

    В A32NX по умолчанию используется следующий вес пассажиров и их багажа.

    • 84 кг для пассажира (включая одежду и ручную кладь)
    • 20 кг для зарегистрированного багажа

    В настоящее время эти веса по умолчанию нельзя изменить в дроне. Планируется предоставить возможность изменить эти значения по умолчанию на странице взвешивания и балансировки flyPad EFB в будущем.

    Можно изменить эти значения по умолчанию, изменив планер simBrief.

    Для этого откройте планер simBrief на веб-сайте simBrief в разделе «МОЙ ФЛОТ», нажав кнопку «Редактировать».

    Прокрутите вниз, чтобы найти записи «Вес пассажира» и «Вес багажа».

    При необходимости измените вес по умолчанию и убедитесь, что вы выбрали «Сохранить самолет» внизу страницы. A32NX будет использовать эти новые значения при следующем импорте рабочего плана полета на основе этого планера.

    Не перегружайте самолет

    Пилот несет ответственность за то, чтобы самолет не был перегружен и находился в установленных пределах.

    Особенно важны:

    • MZFW (максимальный вес без топлива): 64300 кг / 141757 фунтов
    • Максимальная полезная нагрузка (пассажиры, багаж, груз) определяется наименьшим из этих расчетов:
      • MZFW — OEW, или 64300 кг — 42500 кг = 21800 кг
      • MTOW — вес пустого — (топливная загрузка — выруливающее топливо), или 79000 — 42500 — (топливная загрузка — выруливающее топливо)
      • MLW — вес пустого — (все резервное/непредвиденное топливо + дополнительное топливо) или 67400 — 42500 — (все резервное/непредвиденное/дополнительное топливо).
    • При ручной загрузке полезной нагрузки пределы MLW и MTOW в настоящее время не проверяются и находятся в ведении пилота.
    • Simbrief управляет ограничениями полезной нагрузки, автоматически уменьшая количество сумок или пассажиров. Тогда самолет улавливает это правильно.

    Если вы вручную укажете пользовательское количество пассажиров, превышающее ограничения по весу самолета, имейте в виду, что OFP по-прежнему будет отражать выбранное пользователем количество пассажиров.

    Частично это связано с тем, что во время создания OFP количество пассажиров интерпретируется как «забронированные пассажиры», а не текущие пассажиры на борту самолета.

    Загрузка топлива и веса

    Окно MSFS Fuel & Weights на панели инструментов

    Мы заблокировали элементы пользовательского интерфейса в окне MSFS Fuel and Weights. Однако ползунки в окне MSFS подвижны, но в течение 1-3 секунд уровни топлива и полезной нагрузки должны вернуться к исходным значениям.

    ОБРАТИТЕ ВНИМАНИЕ мы изменили загрузку топлива и полезной нагрузки.

    • Топливо: Теперь через EFB.
    • Payload (Dev+Exp): выполняется на вкладке Payload в EFB. См. раздел «Веса и балансировка»
    • .
    • Полезная нагрузка (стабильная): выполнено через W&B в MCDU

    Топливо

    Загрузка топлива теперь осуществляется исключительно через наш EFB, который имеет отличный пользовательский интерфейс для просмотра состояния топливных баков и других параметров. Руководство здесь.

    Гири и балансировка

    Мы представили новую модель полета в сочетании с новым методом взвешивания и балансировки полезной нагрузки, который включает в себя ряды сидений и правильный центр тяжести.

    Пожалуйста, прочтите соответствующее руководство, относящееся к используемой вами версии — стабильной версии или версии для разработчиков.

    Получите наш профиль simBrief.

    Обратите внимание на следующее:

    • В поле грузового отсека теперь отображаются либо метрические тонны, либо тысячи фунтов, в зависимости от единицы измерения, выбранной в настройках EFB для конфигурации самолета.
    • Настоятельно рекомендуется убедиться, что вы выбрали одинаковые веса (килограммы или фунты) в EFB и в OFP/Airframe simBrief перед импортом, чтобы предотвратить любые несоответствия в значениях.

    При запуске дрона из состояния Cold and Dark имейте в виду следующую важную информацию:

    • INIT FUEL PRED необходимо заполнить до вылета. (Это подробно описано в нашем руководстве для начинающих. Убедитесь, что вы выполнили этот шаг при подготовке MCDU.)
    • Если не заполнить страницу INIT FUEL PRED перед запуском двигателя, на блокноте MCDU будет отображаться INITIALIZE WEIGHT/CG желтым цветом.
    • При несоответствии GW более 7 тонн между значениями, рассчитанными аэродинамически, и значением, рассчитанным с использованием введенных ZFW и расхода топлива ПРОВЕРКА ВЕС будет отображаться желтым цветом на блокноте MCDU.

    См. flyPadOS 2 — стабильная версия или flyPadOS 3 — настройки версии для разработчиков страницу, если вы хотите изменить единицу веса, используемую системами самолета.

    Версия для разработчиков

    Топливо, вес и балансировка при запуске в холодную и темную погоду

    Процесс, описанный в этом разделе, предназначен для запуска полета у выхода на посадку в холодном и темном состоянии.

    Если вы начинаете свой полет на взлетно-посадочной полосе или в воздухе, процесс загрузки будет работать только в том случае, если настройки времени посадки в flyPad EFB установлены на Instant . Это сделано преднамеренно, поскольку имитация всего процесса заправки или посадки при старте с взлетно-посадочной полосы не имеет смысла.

    Поиск экрана полезной нагрузки

    Раздел «Полезная нагрузка» можно найти на странице «Наземные службы» на EFB, выбрав вкладку «Полезная нагрузка».

    Оказавшись на этой странице, вы увидите следующее:

    • Модель конфигурации сидений A32NX с видом сверху
    • Поля ввода для настройки PAX и Cargo
    • Запись веса пассажира и веса багажа
    • Настройки времени посадки
    • Динамическая компьютерная графика и диаграмма полезной нагрузки — подробнее см. выше.

    Минимальный посадочный вес (MLDW)

    Это значение будет автоматически заполнено на габаритной диаграмме CG после того, как вы заполните страницу INIT FUEL PRED на MCDU. MLDW представлен 3-й точкой на графике зеленого цвета. Однако, обратите внимание, что это логика-заполнитель, поскольку для более точного прогнозирования расхода топлива потребуются точные прогнозы расхода топлива, которые в настоящее время еще недоступны.

    Конфигурация пассажиров и груза

    Чтобы начать посадку, вы должны сначала ввести значение в поле ввода пассажира или в поле ввода ZFW. Для этого нажмите на соответствующее поле ввода, которое вы хотите изменить, и введите стоимость, которую вы планируете или которая была выделена через ваш OFP. Этот метод является самым простым и позволяет рассчитать соответствующие значения. Конфигурация сиденья A32NX также будет отображаться голубым цветом. места, которые заполнены.

    Пассажир или Поле ZFW

    Важно отметить, что при первом вводе значения в поле Пассажир будет автоматически рассчитан ZFW. И наоборот, если вы сначала вводите значение в Поле ZFW. Пожалуйста, используйте только одно поле для целей планирования.

    Это можно увидеть в примерах ниже:

    Если вы импортировали данные simBrief с главной страницы нашего EFB, вы также можете автоматически заполнить плановые значения для ваших пассажиров и грузов, нажав на кнопку импорта в пользовательском интерфейсе Payload.

    Дополнительные функции

    Дополнительные функции

    Дополнительный груз

    После планирования вашего груза и пассажиров или использования функции импорта simBrief вы можете добавить дополнительный груз, изменив рассчитанное значение груза.

    Убедитесь, что вы изменили это значение ПОСЛЕ ввода значений вашего пассажира или ZFW.

    Грузовые ползунки

    Вы также можете использовать ползунки над каркасом самолета, чтобы заполнить грузовые отсеки. Хотя это может быть не столь точным, это позволяет модифицировать физические распределение груза в самолете. Фактический числовой вес планируемого груза обновится автоматически.

    • Каждый ползунок соответствует одному багажному отделению. В задней части есть перегородка и место для хранения незакрепленных предметов.
    • Белые стрелки на изображении выше показывают, сколько груза нужно заполнить в каждом соответствующем трюме.

    Индивидуальное распределение пассажиров

    Вы также можете взаимодействовать с моделью A32NX, чтобы назначать пассажирские места и влиять на расположение пассажиров на борту.


    ZFW MAC %/CG

    Сейчас этот расчет такой же, как и для обычного CG, но мы не учитываем топливо. Позже мы расширим эту функцию. Для получения дополнительной информации см. Раздел «Центр тяжести» выше.

    Посадка пассажиров

    После того, как вас устроит количество пассажиров и рассчитанный вес, вы можете начать посадку, нажав голубую кнопку «Воспроизвести» в разделе времени посадки. Это поле также позволяет вы можете настроить, как быстро пассажиры входят в самолет.

    После начала посадки кнопка «Играть» станет зеленой, и вы начнете видеть, как пассажиры входят в самолет, значения веса меняются как в динамическом графике, так и в текущие значения раздела конфигурации.

    Сиденья также изменят цвет с темного на светло-голубой, указывая, какие места в данный момент занимают пассажиры.

    Устаревшие инструкции (стабильная версия)
    Устаревшие инструкции (стабильная версия)

    Топливо, вес и балансировка при запуске в холодную и темную погоду

    Процесс, описанный в этом разделе, предназначен для запуска полета у выхода на посадку в холодном и темном состоянии.

    Если вы начинаете свой полет на взлетно-посадочной полосе или в воздухе, процесс загрузки будет работать только в том случае, если настройки времени посадки в flyPad EFB установлены на Мгновенное . Это сделано преднамеренно, поскольку имитация всего процесса заправки или посадки при старте с взлетно-посадочной полосы не имеет смысла.

    Динамические поля и цвета

    Payload, ZFW, ZFWCG — это динамические поля, которые обновляются вместе с процессом загрузки/посадки.

    • Полезная нагрузка = Вес пассажира + Вес багажа + Груз
    • ZFW (масса без топлива) = OEW (эксплуатация пустой массы) + полезная нагрузка
    • ZFWCG = CG на основе ZFW ( Не путать с взлетным CG )

    Станции в ГОЛУБОМ цвете указывают, что они читают/ожидают посадки/загрузки.

    Полностью загруженные станции становятся ЗЕЛЕНЫМИ.

    См. также общую информацию о топливе и весе

    Меню AOC

    Меню AOC (Airline Operational Center) находится в МЕНЮ MCDU и должно использоваться для определенных разделов данного руководства.

    • Нажмите МЕНЮ MCDU
    • Нажмите на ATSU (ATSU = подразделение службы воздушного движения)
    • Нажмите на AOC MENU (AOC = операционный центр авиакомпании)
    • Нажмите на Б/Б
    Информация о загрузке A32NX вручную
    Загрузка вручную

    Можно ввести эти значения вручную, чтобы настроить загрузку пассажиров. При настройке загрузки пассажиров вручную обратите внимание на следующую информацию:

    • Чтобы присвоить значение строке (станции), введите сумму в свой блокнот с помощью клавиатуры MCDU и нажмите соответствующую LSK рядом с нужной станцией.
      • При вводе pax в отдельные строки учитывается только вес пассажира (84 кг или 185 фунтов на пассажира). Багаж и дополнительный груз необходимо добавить отдельно.
    • Если ввести значение в TOTAL PAX с помощью LSK1L, это автоматически распределит пассажиров на основе идеального CG.
      • Вес регистрируемого багажа (20 кг или 44 фунта) добавляется к грузовому отсеку при использовании поля TOTAL PAX .
    • После того, как вышеуказанные веса будут учтены, вы можете ввести остаточный вес (вес груза) в формате X.X , обозначая метрические тонны или тысячи фунтов, в зависимости от того, что единица измерения, выбранная в настройках EFB.
      • Вес груза ограничивается максимальной грузоподъемностью, если он превышает пределы грузового отсека (9435 кг или 20800 фунтов).
    Загрузить информацию о полезной нагрузке OFP

    Оказавшись на странице W&B , она должна выглядеть как пример изображения, даже если вы нажали ЗАПРОС ОФП в другом разделе.

    PAX Rows FormatCargo Hold Format

    • X (Y)
    • X обозначает пассажира, уже посаженного на соответствующую станцию ​​
    • Y обозначает цель PAX (ожидает посадки) для соответствующей станции.
    • X (Y)
    • X обозначает груз, загруженный в трюм
    • Y обозначает грузовую цель для грузового отсека (включая багаж)

    Нажмите OFP Request для этой конкретной страницы, и ваша страница W&B покажет общее количество пассажиров, количество пассажиров в ряду и грузовой отсек, которые заполняются автоматически.

    Примечание: это не запускает процесс посадки. Также груз будет ограничен в качестве защиты до максимальной вместимости, если груз simBrief OFP превышает пределы грузового отсека).

    Борт пассажиров

    Теперь вы можете начать посадку, выбрав LSK6R (индикация изменится с START на STOP желтым цветом) и наблюдать, как пассажиры садятся в самолет.

    Вам не нужно оставаться на этой странице, так как посадка продолжается.

    Перед заполнением ZFW и CG страницы INIT B убедитесь, что посадка и загрузка завершены!

    Установка времени имитации посадки на EFB

    После завершения посадки все ряды и общее количество пассажиров должны стать зелеными, а индикация посадки вернется к START . Проверьте свой ZFW на этом экране и проверьте нижний ECAM, что GW был обновлен.

    Пассажиры на борту
    Полный деборд

    После завершения полета вы можете высадить пассажиров из самолета. Вернитесь в МЕНЮ AOC и перейдите на страницу 2 из 9.1467 W&B .

    Чтобы выполнить полную высадку (все пассажиры и грузы), введите 0 в блокнот и выберите LSK1L в поле TOTAL PAX.

    Нажмите СТАРТ, чтобы начать процесс.

    Частичная деборда

    В случае частичной разгрузки вы можете ввести желаемое количество оставшихся пассажиров и груза для этапа 2. В примере справа показано запланированное удержание 142 пассажиров.

    ПРИМЕЧАНИЕ: Cargo показывает только вес сумки на 142 человека. Пожалуйста, убедитесь, что вы повторно добавили любой дополнительный груз, который вы могли перевозить изначально.

    Нажмите СТАРТ, чтобы начать процесс.


    Стойкость топлива

    Недоступно в стабильной версии

    После завершения полета A32NX сохраняет количество топлива на борту в специальный справочник при выключенном хотя бы одном двигателе. Эта функция привязана к регистрации самолета найдено в atc_id в acft.cfg .

    Если вы решите выйти из симулятора после полета, загрузитесь позже с той же ливреей и соответствующей регистрацией. будет использовать то же количество топлива, что и ваш предыдущий рейс.

Добавить комментарий

Ваш адрес email не будет опубликован. Обязательные поля помечены *